eMedici- surgery Flashcards

1
Q
A

Nonpyogenic liver abscess Amebic liver abscess; Entamoeba histolytica

Hepatic echinococcosis (hydatid cyst of the liver) *

Pyogenic liver abscess- On CT- Atrial phase, double target sign- membranous portion and outer enhancement (odema)

How well did you know this?
1
Not at all
2
3
4
5
Perfectly
2
Q

16 year old male. Knee pain with activity, especially explosive jumping.No swelling of joint. No joint space tenderness. Pain on palpation of anterior tibia (below knee) , pain with resisted extension. Main DDx?

A

Main DDx? Traction apophocitis or Patella tendonitis. Both occur with repetitive, explosive exercises. Key difference: Traction apophocitis- below the knee, at quadriceps tibial tenderness. X-rays are diagnostic for traction apophysitis which would reveal anterior soft tissue swelling and fragmentations of the tubercle. Patella tendonitis- Normal x-ray. anterior knee pain at the inferior border of the patellar and pain with prolonged flexion

How well did you know this?
1
Not at all
2
3
4
5
Perfectly
3
Q

2.8 cm diameter solid lesion in the right kidney, incidental find. 84 yoa with co-morbs

A

This is almost certainly a renal cell carcinoma.Small (<3cm) incidental and asymptomatic lesions in patients over the age of 70 could be managed by active surveillance and left well alone if there is no observed change in size. This woman has some serious co-morbidities and therefore a plan of active surveillance would probably be the most appropriate strategy. Although there are no randomised clinical trials to support this approach, an analysis of the published literature supports this management strategy, showing that active surveillance is safe and feasible, particularly for the elderly and unwell patients. Furthermore, the possibility of developing metastasis from small renal masses is extremely low, supporting the role of active surveillance.A CT-guided biopsy is not indicated; it is unreliable and unable to differentiate between oncocytoma or chromophobe renal cell carcinoma.Partial and total nephrectomies are usually reserved for renal masses 4-7cm and >4cm respectively. Furthermore, they may be inappropriate given this patient’s age and comorbidities.Nivolumab is a programmed death-1 (PD-1) immune checkpoint inhibitor monoclonal antibody. It is a novel treatment for advanced and metastatic renal cell cancer; and would therefore not be appropriate for this case.Small (<3cm) incidental renal masses are best managed with active surveillance in elderly .

How well did you know this?
1
Not at all
2
3
4
5
Perfectly
4
Q

24 hour history of a swelling around his anus, associated with severe pain. This followed an episode of constipation and straining at stool. The lesion is shown (Image).

A

This patient has a prolapsed and thrombosed internal haemorrhoid. There is ulceration over one of the prolapsed haemorrhoids.Thrombosis is a serious complication of internal haemorrhoids. Patients with thrombosed or strangulated haemorrhoids present with severely painful and irreducible, incarcerated haemorrhoids, which may become necrotic. Acutely thrombosed internal haemorrhoids can be extremely painful and distressing. The most effective way of giving this patient rapid relief from his symptoms is to perform a semi-urgent haemorrhoidectomy. Unless the surrounding tissue is necrotic, mucosa and anoderm should be preserved to prevent post-operative anal stricture. If this was a perianal haematoma, then incision and evacuation of the haematoma would be appropriate.Learning PointsProlapsed and thrombosed haemorrhoids are best treated by prompt haemorrhoidectomy.

How well did you know this?
1
Not at all
2
3
4
5
Perfectly
5
Q

5th metatarsal- mid-shaft tenderness, not at base . What #?

A

Stress #- midshaft (diaphysis)Jones #- MetaphysisAvulsion # at base 5th

How well did you know this?
1
Not at all
2
3
4
5
Perfectly
6
Q

A 37-year-old man presents to his general practitioner with a three-month history of an intermittent and painless passage of blood per rectum. This occurs typically when he is constipated. He reports bouts of constipation alternating with normal bowel movements. He has noticed small amounts of blood streaked on the surface of the faeces. Sometimes mucus is present. He has never had these symptoms before. Digital rectal examination is unremarkable. Proctoscopy shows some fleshy haemorrhoids. His blood reports are shown below:Haemoglobin = 107 g/LMCV = 68 fLWBC = 9.1 x 109/LPlatelets = 220 x 109/L

A

Internal haemorrhoids. Blood streaking on the sides of faeces is typically seen in haemorrhoids. Internal haemorrhoids are usually painless while external haemorrhoids are usually painful. Although the haemorrhoids may well be the source of the bleeding - this should never be assumed until other - and potentially more serious - causes have been excluded. Patients with red flags need to undergo further investigations to exclude a potential colorectal carcinoma regardless of their age. Red flags that would indicate further investigation with colonoscopy or sigmoidoscopy in patients with rectal bleeding include:* Change in bowel habits (eg, change in calibre, frequency, and consistency of the stools)* Constitutional symptoms (eg, fever, weight loss, night sweats)* Iron deficiency anaemiaFamily history of colon cancerThis patient has change in bowel consistency and has low MCV (most likely due to iron deficiency anaemia), and could well have a carcinoma of the rectum and a thorough examination of the lower digestive tract must be undertaken - this means a colonoscopy or a flexible sigmoidoscopy. Such a diagnosis is very important to exclude - as colorectal malignancies are now being seen with increasing frequency in younger patients.High fibre diet, injection sclerotherapy, and rubber band ligation are all appropriate management options for haemorrhoids. However, this patient needs to undergo further investigations to rule out colorectal carcinoma.CT abdomen and pelvis may be appropriate for preoperative staging of colorectal cancer to determine the extent of the malignancy. Colonoscopy should be performed first, however, to confirm the diagnosis.Learning PointsColorectal malignancy is now being encountered in younger patients cohorts and this potential diagnosis must always be considered in a patient who presents with rectal bleeding.

How well did you know this?
1
Not at all
2
3
4
5
Perfectly
7
Q

A 66-year-old man presents to the Emergency Department with a four-hour history of sudden onset of severe pain in his right leg. His medical history includes atrial fibrillation, for which he takes apixaban. On examination his blood pressure is 142/84 mmHg and an irregular pulse 100/min. The right leg is pale, cool and no pulses can be felt below the femoral pulse. A CT angiogram show an obstruction at the femoro-popliteal junction. He is diagnosed with a Rutherford class IIa acute limb ischaemia. A heparin infusion is started.Which one of the following is the most appropriate next step in management- various interventions for ALI

A

ALI- need to decide if obstruction d/t embolus or thrombusThrombus- atherosclerosis etcTreated with - angioplasty endovascular ( short segment and aorto-illiac), endarectomy, bypassEmbolus- AF/recent MICatheter directed Thrombolysis with/ w/o embolectomy Bypass procedures might be preferred for thrombotic situations, but in this case with embolism, a catheter-directed line of approach should be considered first. Of course this will depend on the availability of an interventional radiologist. Input from haematology will be important, as the anticoagulation with apixaban will make the risk of intraprocedural bleeding high.Balloon angioplasty and endarterectomy are considered approaches for limb ischaemia caused by a thrombotic phenomenon.

How well did you know this?
1
Not at all
2
3
4
5
Perfectly
8
Q

ALI mx of each class

A

Acute limb ischaemia is often characterised by a sudden onset of these symptoms. A normal, pulsatile contralateral limb is a sensitive sign of an embolic occlusion.In the history, the causes of potential embolisation should be explored. These include chronic limb ischaemia, atrial fibrillation, recent MI (resulting in a mural thrombus), or a symptomatic AAA (ask about back/abdominal pain) and peripheral aneurysms.The later the patient presents to a hospital, the more likely that irreversible damage to the neuromuscular structures will have occurred (more common >6hrs post-symptom onset), which will ultimately result in a paralysed limb.CategoryPrognosisSensory LossMotor DeficitArterial DopplerVenous DopplerI – ViableNo Immediate threatNoneNoneAudibleAudibleIIA – Marginally ThreatenedSalvageable, if promptly treatedMinimal (toes) or noneNoneInaudibleAudibleIIB – Immediately ThreatenedSalvageable if immediately revascularisedMore than toes, rest painMild/ModerateInaudibleAudibleIII – IrreversibleMajor tissue loss, permanent nerve damage inevitableProfoundProfound, paralysisInaudibleInaudibleTable 1 – Clinical Categories of Acute Limb Ischemia, adapted from Rutherford et al., 2009

How well did you know this?
1
Not at all
2
3
4
5
Perfectly
9
Q

allergic rhinitis presents more with symptoms of

A

sneezing, rhinorrhoea, watery eyes, and if chronic patients may also complain of postnasal drip, chronic nasal congestion, and obstruction. It can be intermittent, with particular triggers or exposures.

How well did you know this?
1
Not at all
2
3
4
5
Perfectly
10
Q

Amebiasis

A

Entamoeba histolytica, a protozoanTransmissionFecal-oral* Amebic cysts are excreted in stool and can contaminate drinking water or foodInfection typically occurs following travel to endemic regionsIncubation and Clinical Features Intestinal amebiasis: 1–4 weeksExtraintestinal amebiasis: a few weeks to several yearsClinical coursesIntestinal amebiasis (Amebic dysentery)Loose stools with mucus and bright red bloodPainful defecation, tenesmus, abdominal painFever in 10–30%* High risk of recurrenceAlways consider amebiasis when a patient presents with persistent diarrhea after traveling to a tropical or subtropical destination!Extraintestinal amebiasis* Mostly acute onset of symptoms; subacute courses are rareIn 95% of cases: amebic liver abscess, usually a solitary abscess in the right lobeFever in 85–90%RUQ pain or pressure sensationDiarrhea precedes only a third of all cases of amebic liver abscesses.Intestinal amebiasisStool analysisMicroscopic identification of cysts or trophozoites in fresh stoolThe following tests confirm the microscopic findings (important since E. histolytica and Entamoeba dispar are morphologically identical ):EIA or coproantigen ELISAPCRStool microscopy is not sensitive; at least three stool samples should be examined before reporting a negative resultExtraintestinal amebiasisSerological antibody detectionAspiration of abscesses: shows brown fluid/pus (exudate resembles anchovy paste)In amebic hepatic abscessALP, AST, ALT, bilirubin slightly elevatedImaging: shows a solitary lesion, typically in the right lobe of the liverTreatmentMedical therapyAsymptomatic intestinal amebiasisIn nonendemic areasLuminal agents such as paromomycin, diloxanide, or iodoquinolSymptomatic intestinal amebiasis and invasive extraintestinal amebiasisInitial treatment with a nitroimidazole derivative such as metronidazole or tinidazoleFollowed by a luminal agent (e.g., paromomycin, diloxanide, or iodoquinol) to eradicate intestinal cysts and prevent relapseInvasive proceduresAspiration: of complicated liver abscesses at risk for perforationSurgical drainage: should generally be avoided

How well did you know this?
1
Not at all
2
3
4
5
Perfectly
11
Q

An ultrasound confirms the presence of gallstones. She is counselled to undergo a laparoscopic cholecystectomy.What is most suggestive that an ERCP should be undertaken prior to the cholecystectomy?

A

In the assessment of any patient with symptomatic gallstones, consideration must always be given to the chances of finding stones in the common bile duct (CBD), which occurs in 10-20% of patients. If stones in the common bile duct are identified on US, then further imaging is not usually required and these patients should proceed to ERCP pre-operatively for stone extraction or laparoscopic cholecystectomy with bile duct exploration (if surgical expertise permits). Studies have found these approaches to be equally valid. ie Stones visable- ERCP (less sensitive MRCP) will suffice. Patients who have deranged liver function tests (LFTs) with normal calibre biliary system, or CBD dilation with normal LFTs should either have further imaging with MRCP, or proceed to laparoscopic cholecystectomy with intraoperative cholangiogram (IOC) for ductal imaging. IE MRCP is more sensitive. Some moderate elevation of all serum liver enzymes can occur after episodes of biliary colic or acute cholecystitis. The finding of a common bile duct greater than 7mm might increase the probability of a stone in the duct - but this diameter can still be a normal finding in many individuals.

How well did you know this?
1
Not at all
2
3
4
5
Perfectly
12
Q

anterior knee pain that worsens with physical activity, involving knee extensions (jumping or lunges). Proximal tibial swelling involved as well. X-ray- anterior soft tissue swelling and fragmentations of the tubercle. Disease name

A

Traction apophysitis, also known as Osgood-Schlatter disease, is a tibial osteochondritis at the tibial insertion of the quadriceps tendon. It is thought to be due to the overuse of quadriceps muscle during growing teenagers or children. It is more common in boys than in girls. The overuse of the quadriceps, especially during physical activities involving jumping or sprinting, during ossification periods in a child may cause repeated avulsion of the patellar ligament on the tibial tuberosity. This results in inflammation or irritation at the apophysis.Clinical features involve * anterior knee pain that worsens with physical activity,* physical activity commonly involving knee extensions such as jumping or lunges.* There may be proximal tibial swelling involved as well.X-rays are diagnostic for traction apophysitis which would reveal: * anterior soft tissue swelling and fragmentations of the tubercle.Treatment involves non-operative management such as resting, ice and NSAIDs for pain relief. Cast and immobilisation for 6 weeks or ossicle excision may be necessary if there are severe symptoms involved. This disease may be self-limiting but may not resolve until bone growth has halted.

How well did you know this?
1
Not at all
2
3
4
5
Perfectly
13
Q

Bariatric surgery: types of procedures with specific C/I

A

Some general contraindications to consider for any bariatric surgery include * severe heart failure,* unstable heart disease,* end-stage lung disease,* cancer or liver disease and* drug/alcohol addiction/dependence.Additionally, the surgery requires general anaesthetic and hence the patient must be suitable for this. A contraindication to sleeve gastrectomy is gastro-oesophageal reflux disease (GORD), as this type of bariatric procedure can worsen the reflux and increase the risk of Barrett’s oesophagus. Sleeve gastrectomy is a procedure to remove a large portion of the stomach (around the greater curvature) to reduce the size of the stomach. As can be imagined with a reduced stomach capacity reflux worsens in these individuals as there is both less room in the stomach for acidic contents to go as well as reduced cells able to produce bicarbonate and other anti-reflux products.Roux-en-Y gastric bypass surgery involves reducing the size of the stomach to a small pouch and connecting this directly onto the small intestine. This procedure has been determined to be the most effective bariatric surgery in terms of both weight loss and improvement of reflux symptoms. Hence, if he were to undergo a type of bariatric surgery this would be the ideal procedure for him. Compared to sleeve gastrectomy which has been shown to worsen already existing disease and even increase the incidence of disease in previously unaffected individuals. Active renal calculi would be a factor in delaying elective surgery such as bariatric, but his history of renal calculus disease two years prior does not influence his suitability for the surgery now. Undergoing the surgery can place the body under stress and cause dehydration, which can precipitate stone formation, so this may be taken into consideration by the anaesthetist.Learning PointsGORD is a contraindication for a sleeve gastrectomyIn a patient with GORD Roux-en-Y gastric bypass is the best option for bariatric surgeryThe indications for bariatric surgery include a BMI >40 or BMI >35 + co-morbid conditions

How well did you know this?
1
Not at all
2
3
4
5
Perfectly
14
Q

Benign breast lessions

A

Fibrocystic changesInflammatory breast conditions* Mastitis* Breast abscess* Fat necrosis of the breast* Mammary duct ectasiaBenign breast neoplasms* Fibroadenoma- young women- mousy* Phyllodes tumor- 40-50. Varied growth U/S similar to Fibroad. Tx is to excise (benign, but Ca equivalent difficult to distinguish)Intraductal papilloma- 40-50 y/o, Intraductal papilloma is the most common cause of bloody nipple discharge. U/S, mamog; Core biopsie if palpable. Tx excison of duct. Lobular carcinoma in situ (LCIS)- malignant trans to invasive less than DCIS. ore biospy and Immunohist diff DCIS/LCIS. * Follow up imaging for LCISExcision for non classic LCIS Mastalgia - Cyclical and non cyclical

How well did you know this?
1
Not at all
2
3
4
5
Perfectly
15
Q

carcanoid tumour in removed appendix (<10mm and clear margins)

A

Appendiceal carcinoid tumours that are less than 10mm in size and with clear margins do not require any further follow up or investigations. Carcinoid or neuroendocrine tumours can be found incidentally in up to 2% of appendicectomy specimens. Small (<10mm) carcinoid tumours with clear margins that are found incidentally at the time of appendicectomy do not need any further treatment or follow-up.Urinary 5-hydroxyindoleacetic acid (5HIAA) is a serotonin metabolite which may be raised in patients with carcinoid tumours. However, this test should only be used in patients who are symptomatic and is not required especially as the tumour has been surgically removed. Serum chromogranin is a carcinoid tumour marker and is usually normal in patients with a tumour size of <2cm. It is not required for the follow up of this patient. CT abdomen is not required for this patient due to the tumour size (<10mm) and clear margins. A right hemicolectomy may be required for patients with more advanced tumours or residual disease.

How well did you know this?
1
Not at all
2
3
4
5
Perfectly
16
Q

choice of chemo prophalaxis in non orthopeadic surgery

A

Low molecular weight heparin is the agent of choice for VTE prophylaxis based on direct data showing efficacy in nonorthopaedic surgical populations. Unfractionated heparin is an alternative in patients with renal insufficiency. Oral agents such as apixaban have limited evidence in nonorthopaedic populations, Aspirin is an anti-platelet and is not indicated.

How well did you know this?
1
Not at all
2
3
4
5
Perfectly
17
Q

Clinical features of meniscal tearsTests for meniscal

A

Knee pain: exacerbated by weight‑bearing or physical activityJoint line tenderness (medial or lateral)Restricted knee extensionIntermittent joint effusionsMcMurray test [1]The patient lies in a supine position.The examiner holds the patient’s knee in one hand and palpates the joint spaces while holding their ankle in the other.The examiner brings the patient’s knee to maximal flexion.For medial meniscus tear, the examiner performs external rotation of the tibia and applies valgus stress while extending the knee.For lateral meniscus tear, the examiner performs internal rotation of the tibia and applies varus stress while extending the knee.Pain on palpationPalpable or audible pop/click with maneuvers

How well did you know this?
1
Not at all
2
3
4
5
Perfectly
18
Q

Clinical features to distinguish lesser and greater trochanteric #

A

A greater trochanter fracture is suggested by local pain exacerbated by abductionA lesser trochanter fracture presents with groin pain, which radiates to the knee or posterior thigh, and worsens with hip flexion and rotation

How well did you know this?
1
Not at all
2
3
4
5
Perfectly
19
Q

Clinical findings of osteoarthritis

A

Common clinical findings: Pain during or after exertion (e.g., at the end of the day) that is relieved with restPain in both complete flexion and extensionCrepitus on joint movementJoint stiffness and restricted range of motionMorning joint stiffness usually lasting < 30 minutesJoint-specific findings Heberden nodes; pain and nodular thickening on the dorsal sides of the distal interphalangeal jointsBouchard nodes: pain and nodular thickening on the dorsal sides of the proximal interphalangeal jointsHeberden and Bouchard nodesIn contrast to rheumatoid arthritis, osteoarthritis can affect the distal interphalangeal joints.

How well did you know this?
1
Not at all
2
3
4
5
Perfectly
20
Q

complication of pancreatitis- necrotising pancreatitis (. CT non enhancement with peri-pancreatic stranding). Unwell- fever signs of sepsis (bloods), When is the risk of infection of necrotic pancreatitis greatest? What investigation is appropriate to initiate?

A

Per-cutaneous aspiration, via FNA then culture and microscopy.This patient has developed a complication of acute pancreatitis - namely, pancreatic necrosis. This local complication puts this man in the category of moderately-severe acute pancreatitis. One-third of patients with necrotising pancreatitis will develop infection within the necrotic tissues. Typically such infection occurs 7-10 days after the onset of pancreatitis and this will manifest by the patient becoming unwell with fever and showing evidence of sepsis. If infection is present the patient will require antibiotic therapy, however percutaneous or endoscopic drainage risks introduction of infection into a sterile field. Therefore, needle aspiration is more prudent, with aspirate being sent for laboratory analysis and culture. If required, drain insertion can be US or CT guided. If the aspiration proves to be aseptic, then antibiotics are not required as use of antibiotics to prevent infection is not supported.If the culture is positive or his condition deteriorates then antibiotics may be initiated. Initial percutaneous drainage prevents the greater risk of complications associated with surgical debridement (necrosectomy), while maintaining the ability to step-up treatment if there is inadequate response. DDX Walled-off necrosisDefinitionAn encapsulated collection of sterile necrotic material, usually occurring > 4 weeks after the onset of necrotizing pancreatitis [5]Previously known as pancreatic abscessDiagnostics: CT abdomen with IV contrast showing an encapsulated heterogeneous collection containing fluid and debris [8]Treatment (of symptomatic walled-off necrosis): percutaneous drainage or transmural endoscopic necrosectomyNB pancreatic psuedocyst is a feature of acute and chronic pancreatitisNecrotizing pancreatitis [14]Definition: necrosis of pancreatic and peripancreatic tissueClinical features: fever, persistent tachycardia, or insufficient symptomatic improvement over several daysDiagnostics: nonenhancing areas of pancreatic parenchyma on CECT abdomen [8]Treatment [14]Sterile necrotizing pancreatitis can usually be managed conservatively. [6]Encourage enteral nutrition if feasible.* Provide supplemental nutritional support as needed. Infected necrotizing pancreatitis [14]Definition: bacterial superinfection of necrotic pancreatic parenchymaClinical features: similar to those of necrotizing pancreatitisDiagnosticsLaboratory studies: persistent or worsening leukocytosis, bacteremia, increasing inflammatory markers [14]CECT abdomen: gas within the pancreas and/or peripancreatic tissue or fluid collections [5]Fine-needle aspiration of necrotic areas: not routinely recommended [6][10]Treatment [14]Supportive care: fluid therapy, analgesics, nutritional supportBroad-spectrum empiric antibiotics with good tissue penetration (e.g., carbapenems ) for 4 weeks [14]Drainage of infected material if there is clinical deterioration or persistence of symptoms despite antibiotic therapyOperative pancreatic debridement (necrosectomy) should ideally be performed at least 2–4 weeks after initial presentation. [14]Minimally invasive procedures (e.g., image-guided percutaneous drainage) can be performed in the first 2 weeks in seriously ill patients.Prognosis: high mortality rate (30%) [14]

How well did you know this?
1
Not at all
2
3
4
5
Perfectly
21
Q

Contrast features of cyclical and non-cyclical Mastalgia. What is imaging and tx for each

A

Cyclical mastalgiaOften bilateral, diffuse breast painTypically, most severe in the upper outer quadrant of the breastsMay radiate to the medial aspect of the upper armUsually worsens the week prior to the onset of menstruationNoncyclical mastalgiaUnilateral or bilateral breast pain, usually located over the costal cartilages* Sharp or burning pain and/or sorenessDiagnostics* Medical history (e.g., hormone therapy, trauma, surgical history, risk factors for breast cancer)* Physical examination: focused breast examination* Look for signs of infection (e.g., erythema, swelling, pain)* Rule out signs suggestive of breast malignancy (e.g., skin changes, mass, nipple discharge)Imaging* Breast ultrasound and/or mammographyIndications: depend on the patient’s age and the presence of findings suggestive of malignancyWomen with cyclical breast pain usually do not require imaging.Women with noncyclical or focal breast pain that is not extramammary should undergo breast imaging.< 30 years of age: ultrasound* 30–39 years of age: ultrasound and/or mammography* ≥ 40 years of age: ultrasound and/mammographyTreatmentFirst-line treatment: conservativeProvide reassuranceRecommend well-fitting sports braUse of warm or cold compresses* Analgesia (e.g., acetaminophen, NSAIDs)Second-line treatment: for patients with persistent (> 6 months of conservative treatment) or severe symptoms* Tamoxifen* Postmenopausal hormone therapy should be decreased or discontinued if it is the cause of breast pain.

How well did you know this?
1
Not at all
2
3
4
5
Perfectly
22
Q

CT findings would be the strongest reason for admission to hospital?A 5 mm stone in the lower ureterA stone at the pelvi-ureteric junctionStranding around the right kidneyA pelvic kidneyMild hydronephrosis

A

Peri-renal stranding seen on CT suggests pyelonephritis.The finding of stranding around the kidney, in addition to a clinical picture of fever and renal angle tenderness, would support a diagnosis of pyelonephritis and the need for antibiotics and inpatient urological consultation for consideration of lithotripsy or stenting (the latter if the kidney appeared to be obstructed). Whilst a solitary kidney might well be an indication for inpatient treatment of nephrolithiasis, one situated in the pelvis should not influence management. Mild hydronephrosis alone is not necessarily an indication for inpatient treatment, but should be considered with other factors such as stone size, location, and other co-morbidities when deciding on a management plan.An uncomplicated 5mm stone is not an indication for inpatient treatment. If the stone does not pass spontaneously, the patient may be referred for an outpatient urological opinion. A stone at the pelviureteric junction (PUJ) is not an indication for inpatient treatment, unless it was causing obstruction. Depending on stone size and composition, many of these stones at this site (PUJ) may pass spontaneously and those that do not, may be treated on an outpatient basis.

How well did you know this?
1
Not at all
2
3
4
5
Perfectly
23
Q

ddx for new nuerology post anuerysmal SAH

A

differentials include complications of the haemorrhage, such as vasospasm (subsequently causing delayed cerebral ischaemia), hydrocephalus, re-bleeding, and seizure, as well as intercurrent problems such as hyponatraemia, sepsis, metabolic encephalopathy, and de novo ischaemic stroke. The non-localising features seen (confusion, reduced conscious level, headache) are common across these differentials, but the superimposed localising features (left-sided faciobrachial weakness) increase suspicion of a territorial vascular event. risk 3-14 days post anuerysmal SAH In this case, the most likely cause is delayed cerebral ischaemia.Re-bleeding is a catastrophic complication of untreated ruptured aneurysms that typically occurs early: a third within 3 hours, half within 6 hours. Overall, 15-20% re-bleed within 2 weeks. Hydrocephalus is detected on initial imaging in ~15-20% of SAH patients, caused by extravasated blood products interfering with CSF circulation. Only 3% of patients without hydrocephalus on initial imaging go on to develop it within the first week. Up to a quarter of SAH patients have seizures at onset, and a third have delayed seizures; non-convulsive status epilepticus occurs in 3-18%. De novo thromboembolic stroke would be quite coincidental and is unlikely (n.b. the next step of workup for this patient - a CT brain with angiography and perfusion maps - is the same scan used to investigate a potential stroke and would identify this anyway).

How well did you know this?
1
Not at all
2
3
4
5
Perfectly
24
Q

DDX for shoulder injuries

A

AC joint disruption would be more likely to present with obvious deformity of the acromioclavicular joint of the shoulder, usually following some form of traumatic event (e.g. direct trauma to the shoulder)Cervical radiculopathy often presents in older patients, who are at greater risk of osteoarthritic degenerative changes, which may subsequently cause foraminal stenosis and nerve root impingement. Furthermore, this condition often presents with paraesthesia or anaesthesia in C5-T1 dermatomal distributions, alongside weakness in accompanying myotomal movements.Bicepital tendonitis is more likely to present with tenderness over the long head of biceps brachii, alongside some potential swelling over the same area.Subacromial bursitis is perhaps the most reasonable differential for rotator cuff tear, however this would present with * a painful abduction arc, * and positive Neer’s (flexion to >90 degrees causing pain) and* Hawkin’s-Kennedy (internal rotation and flexion to 90 degrees causing pain) tests’.

How well did you know this?
1
Not at all
2
3
4
5
Perfectly
25
Q

ddx of groin mass

A

* Rectus sheath haematoma* Indirect inguinal hernia* Femoral hernia* Direct inguinal hernia* Saphena varixpatient on warfarin, a rectus sheath haematoma is a reasonable differential for an abdominal mass with pain. Other findings may include a history of trauma, haemodynamic instability and abdominal bruising. These haematomas are typically deep to the rectus abdominis muscle and do not extend into the groin.A saphena varix occurs due to dilation of of the saphenous vein at the saphenofemoral junction. This is may result in a palpable groin mass and will typically be associated with varicose veins elsewhere in the limb. These swellings are soft.Learning PointsIndirect inguinal hernias pass through the inguinal ring

How well did you know this?
1
Not at all
2
3
4
5
Perfectly
26
Q

84 year old with co-morbidities2.8 cm lesion on the right kidney was incidentally found. Her renal function is normal and there is no evidence of metastatic disease on further CT scanning

A

There is a 2.8 cm diameter solid lesion in the right kidney. This is almost certainly a renal cell carcinoma.Small (<3cm) incidental and asymptomatic lesions in patients over the age of 70 could be managed by active surveillance and left well alone if there is no observed change in size. This woman has some serious co-morbidities and therefore a plan of active surveillance would probably be the most appropriate strategy. Although there are no randomised clinical trials to support this approach, an analysis of the published literature supports this management strategy, showing that active surveillance is safe and feasible, particularly for the elderly and unwell patients. Furthermore, the possibility of developing metastasis from small renal masses is extremely low, supporting the role of active surveillance.Partial and total nephrectomies are usually reserved for renal masses 4-7cm and >4cm respectively. Furthermore, they may be inappropriate given this patient’s age and comorbidities.

How well did you know this?
1
Not at all
2
3
4
5
Perfectly
27
Q

Dermatomes of cerval spine- ie loss of sensation

A
How well did you know this?
1
Not at all
2
3
4
5
Perfectly
28
Q

difference between pancreatic psuedocyst vs walled off necrosis

A

Pancreatic pseudocysts Encapsulated collection of pancreatic fluid that develops 4 weeks after an acute attack of pancreatitis (can occur in both acute and chronic pancreatitis) [11]PathophysiologyPancreatic secretions leak from damaged ducts → inflammatory reaction of surrounding tissue → encapsulation of secretions by granulation tissueClinical features [26]* Often asymptomatic* Painless abdominal massPressure effectsGastric outlet obstruction (early satiety, nonbilious vomiting, abdominal pain)Obstruction of the distal duodenum (bilious vomiting) may result in steatorrhea.Bile duct obstruction with jaundiceDiagnostics [27]First line: CT abdomen with contrastFindings: Extrapancreatic fluid collection within well-defined wall or capsule with contrast enhancementOther imaging modalitiesTransabdominal ultrasound* Fast and readily availableHigh sensitivity, but low negative predictive valueERCP (gold-standard test): more invasive, but allows treatment to be performedMRI/MRCP: highly sensitive and specific test, but associated with high costWalled-off necrosisDefinitionAn encapsulated collection of sterile necrotic material, usually occurring > 4 weeks after the onset of necrotizing pancreatitis [5]Previously known as pancreatic abscessDiagnostics: CT abdomen with IV contrast showing an encapsulated heterogeneous collection containing fluid and debris [8]Treatment (of symptomatic walled-off necrosis): percutaneous drainage or transmural endoscopic necrosectomy

How well did you know this?
1
Not at all
2
3
4
5
Perfectly
29
Q

do displaced or non- displaced intracapsular NOF # need arthroplast (ie head of femor replacement)

A

Neck of femur (NOF) fractures can occur anywhere from the subcapital region of the femoral head to 5cm distal to the lesser trochanter (Fig. 1).By TeachMeSeries Ltd (2020)Figure 1 – The bony landmarks of the anterior proximal femurThe neck of femur can be considered to have two distinct areas, which are described relative to the joint capsule:Intra-capsular – from the subcapital region of the femoral head to basocervical region of the femoral neck, immediately proximal to the trochantersExtra-capsular – outside the capsule, subdivided into:Inter-trochanteric, which are between the greater trochanter and the lesser trochanterSub-tronchanteric, which are from the lesser trochanter to 5cm distal to this pointThe blood supply to the neck of the femur is retrograde*, passing from distal to proximal along the femoral neck to the femoral head. This is predominantly through the medial circumflex femoral artery, which lies directly on the intra-capsular femoral neck.Consequently, displaced intra-capsular fractures disrupt the blood supply to the femoral head and, therefore, the femoral head will undergo avascular necrosis (even if the hip is fixed). Patients with a displaced intra-capsular fracture therefore require joint replacement (arthroplasty), rather than fixation.Intracapsular fractures can also be further classified by the Garden Classification (Table 1)Garden ClassificationSimplified ClassificationDescriptionINon-displacedIncompleteIIComplete fracture but nondisplacedIIIDisplacedComplete fracture, partial displacementIVComplete fracture fully displaced

How well did you know this?
1
Not at all
2
3
4
5
Perfectly
30
Q

DOC for recently treated billary colic .

A

Tramadol 50mg immediate release oral tablet is the most appropriate choice for analgesia in these circumstances. Codeine has a spasmodic effect on the sphincter of Oddi, and so should be avoided in biliary colic. It was even associated with acute pancreatitis in patients with history of cholecystectomy (odds ratio 2.64) in a large case controlled study.Slow release opioids and patches are inappropriate for the treatment of acute pain. A 2018 position statement from the Australian and New Zealand College of Anaesthetists: ‘the inappropriate use of slow-release opioids for the treatment of acute pain has been associated with a significant risk of respiratory depression, resulting in severe adverse events and deaths.’Learning PointsTramadol (IMMEDIATE RELEASE) provides effective pain relief for recently treated biliary colic.

How well did you know this?
1
Not at all
2
3
4
5
Perfectly
31
Q

ecently noticed some discomfort in his lower limbs, particularly towards the end of the day. On examination, his BMI is 34 and his blood pressure is 146/86. All other vital signs are normal. Systemic examination is largely unremarkable, although the following changes are observed on inspection of the right lower limb (Image). The legs are non-tender, with a normal temperature gradient.

A

A term used to describe the edema, characteristic skin changes (hyperpigmentation, stasis dermatitis), and ulcers secondary to chronic venous hypertension. Varicose veins may or may not be present. This patient presents with lower limb discomfort in the setting of an elevated BMI. This information, along with the photograph, which demonstrates haemosiderin staining, suggests chronic venous insufficiency to be the correct answer. The mechanism behind the haemosiderin staining in venous skin changes is due to venous hypertension (due to vein incompetence or occlusion) resulting in increased capillary permeability with resultant tissue oedema, leakage of red cells (breaking down to haemosiderin) and extravasation of activated white cells, leading to an inflammatory cascade.Necrobiosis lipoidica is a rare granulomatous lesion that is most often observed on the shins of diabetic patients. Initially, this condition begins as a dull red papule or plaque on the shin which slowly enlarges into one or more yellowish-brown patches with a red rim. Whilst this patient does have diabetes, the appearance of the leg as seen in the photograph is not consistent with necrobiosis lipoidica. As such, this is not the correct answer. Features of venous hypertensionGeneralized or localized pain, lower extremity discomfort/cramping, and limb swelling* Worsened by heat* Worse while standing, relieved by walking and raising of legsPruritus, tingling, and numbnessEdema (may be unilateral) that starts in the ankle and may involve the calf later in the disease course (in about half of affected individuals)TelangiectasiasYellow-brown or red-brown skin pigmentation of the medial ankle; later of the foot and possibly lower legRBC breakdown leads to hemosiderin release → accumulation in the dermis → skin pigmentationMay lead to stasis dermatitis; a scaly, pruritic rashParaplantar varicose veinsLipodermatosclerosis: Localized chronic inflammation and fibrosis of skin and subcutaneous tissues of lower leg [7]Painful, indurated, and hardened skinAtrophie blanche: white, coin- to palm-sized atrophic plaques due to absent capillaries in the fibrotic tissue

How well did you know this?
1
Not at all
2
3
4
5
Perfectly
32
Q

elderly patient with streptococcus bovis infection. What occult malignancy should be suspected. Which organisms are associated with cholecystitis?

A

Strep Bovis is associated (strongly) with colorectal cancer. Colonoscopy should be arranged ASAP. Treatment- Cephazolin. S.Bovis can cause- Infect Endo and CholecystitisS. Bovis is not a usual organism associated with Cholecystitis. Normal orgs (gram - KEEPS)Klebsiella pneumoniae, Escherichia coli, Enterobacter cloacae, Proteus vulgaris and Streptococcus faecalis. Empirical antibiotic therapy is aimed at this with metronidazole and ceftriaxone; another antibiotic that may be added is gentamicin.

How well did you know this?
1
Not at all
2
3
4
5
Perfectly
33
Q

epitaxis in an elderly patient with AF and anti coagulated, ongoing for a while

A

ABCDE_ Airway protection, assesment and large bore IV first!!!!The majority of episodes cease with simple nasal compression, however, the clinician must be aware of the potential for catastrophic bleeding. Appropriate assessment is needed in this patient as she is on anticoagulation and is elderly. As such a Primary Survey approach is most suitable, starting with airway assessment as blood clots can occlude the airway. Insertion of a wide-bore cannula is a standard practice in resuscitation; in this case, although the patient is currently haemodynamically stable, she may quickly decompensate and require aggressive volume expansion. Compression of the nose can be useful but the bleeding has been going for an hour and there is more pressing management. Silver nitrate cauterisation may be beneficial if there is a clear identify anterior bleeding vessel but AFTER an primary survey assessment. ENT are the experts in managing these scenarios and should be called after initial management is commenced. Radiological embolisation is a management option after considering the above.Learning PointsEpistaxis can be an emergency and requires A-E assessment

How well did you know this?
1
Not at all
2
3
4
5
Perfectly
34
Q

Features of tension headche

A

This is because a tension headache typically is located in a ‘band’ around the head, holocranial, or bifrontal, and is not usually located in the occiput. It can also be associated with a maximum of one of the following symptoms – nausea, phonophobia, or photophobia.

How well did you know this?
1
Not at all
2
3
4
5
Perfectly
35
Q

first line and scond line Rx for cyclical mastalgia

A

First-line therapy for cyclical mastalgia pain includes conservative measures including reassurance, physical aids (clothing, etc), and simple analgesia.Second-line therapies for cyclical mastalgia may be required in patients who continue to have severe breast pain for six months despite first-line therapy.Medications used to treat mastalgia have a significant side effects and their use needs to be strictly monitored by a GP or in consultation with a specialist. A trial of pharmacologic therapy may be considered when patients have severe pain or when conservative measures fail.First-line therapy for cyclical breast pain involves conservative measures including reassurance, breast support (ie., a well-fitting bra), and simple analgesia like nonsteroidal anti-inflammatories (i.e., Panadol or additional NSAIDs). Topical NSAIDs can also be used (i.e., diclofenac gel).Second-line therapy for cyclical mastalgia is indicated with consistent pain for 6-months despite adequate first-line therapy. Options include Tamoxifen and Danazol.Tamoxifen use is restricted due to adverse effects include thromboembolism and endometrial cancer. Further, to this, as an aromatase inhibitor, it can also induce vaso-motor symptoms, menstrual irregularity, vaginal discharge, hair thinning, and weight gain. The patient is 35-years-old, and premenopausal, good practice should also include advice about the risk of reducing her bone density, and the implementation of strategies to maintain bone health. Danazol is an androgen that can cause weight gain, oily skin, hot flashes, menstrual irregularities, and virilization. It is contraindicated in pregnancy and has been identified as a risk factor for arterial thrombosis. Given these many undesirable effects, both tamoxifen and danazol should be used under the guidance of a GP, OBGYN, or appropriate specialty.

How well did you know this?
1
Not at all
2
3
4
5
Perfectly
36
Q

First line investigation of suspected prostate Ca

A

Multi-parametric MRI is now becoming the standard of practice in the initial assessment of a patient with suspected carcinoma of the prostate.

How well did you know this?
1
Not at all
2
3
4
5
Perfectly
37
Q

First step of MX of osteoarthritis.

A

Learning PointsMuscle strengthening exercises should be the first step in the management of osteoarthritis.Over the counter pain relief such as paracetamol may provide some benefit to patients. Prolonged courses of non-steroidal anti-inflammatories should be avoided however, especially in patients with renal impairment.Mainstays of treatment include weight loss if BMI is elevated, and physical therapy to improve joint stability through muscle strengthening exercises. Intra-articular injections have also been shown to confer benefit in OA patients, however in a randomised controlled trial of both treatment modalities, patients who underwent physical therapy had less pain and functional disability at 12 months without the adverse effects associated with pharmacological interventions.Curcumin is advocated as an anti-inflammatory herbal preparation, but is of unproven value.

How well did you know this?
1
Not at all
2
3
4
5
Perfectly
38
Q

For a unilateral joint pain and swelling, reasonable differentials would include septic arthritis, crystal arthropathy, haemarthrosis, fracture, and soft tissue injury.

A
How well did you know this?
1
Not at all
2
3
4
5
Perfectly
39
Q

Gardner classification for femur neck #

A

Garden classification of femoral neck fracturesThe degree of femoral neck displacement is determined by the course of the trabeculations in the femoral neck and head (depicted here as blue lines; grey area = impaction; red area = line of fracture).Garden I: nondisplaced, incomplete, impaction fracture with valgus configuration (lateral distortion of the trabeculations)Garden II: complete, but nondisplaced fracture (disruption of the trabeculations without distortion)Garden III: complete and partially displaced fracture with medial contact of the fracture elements and varus displacement (medial distortion of the trabeculations)Garden IV: complete, fully displaced fracture

How well did you know this?
1
Not at all
2
3
4
5
Perfectly
40
Q

GIST vs Gastric Cancer

A

gastrointestinal stromal tumour (GIST). Large mass in stomachOn CT This is because of the well-circumscribed, homogenous mass in the stomach surrounded by contrast. These tumours usually occur after 40 years of age, and most commonly occur in the stomach. They are most likely to be asymptomatic, and are hence commonly discovered incidentally upon CT scan or endoscopy. Depending on their location in the digestive tract, GISTs are unlikely to cause symptoms until they are very large. If large, they can ulcerate and sometimes cause gastrointestinal haemorrhage. They also obstruct or may present with non-specific gastrointestinal symptoms. Gastric carcinoma may have no specific symptoms when they are superficial and potentially curable, although some patients have nonspecific gastrointestinal complaints such as dyspepsia. As it progresses they may present with anorexia, weight loss, as well as vague abdominal pain. If this lesion was a gastric cancer, it would almost certainly have produced symptoms by now.

How well did you know this?
1
Not at all
2
3
4
5
Perfectly
41
Q

Hip fractures are divided into:

A

IntracapsularFemoral headFemoral neck (sub capital) ExtracapsularTrochantericIntertrochantericSubtrochantericTypical fracture sites of the proximal femur

How well did you know this?
1
Not at all
2
3
4
5
Perfectly
42
Q

How is a suspected CSF leak tested for?

A

The presence of CSF can be confirmed by testing the fluid for beta-2-transferrin.

How well did you know this?
1
Not at all
2
3
4
5
Perfectly
43
Q

Hydatid cyst

A

Hydatid cysts are parasitic infections typically located within the liver, occurring due to ingesting food contaminated with E. granulosus eggs. The domestic dog is the main host of E. granulosus. They present on CT as well-defined encapsulated cystic or multicystic masses. Pathogens: Echinococcus tapewormsEchinococcus granulosus causes CEEchinococcus multilocularis causes AETransmissionHand-to-mouth* petting a dog or catContaminated dirt (e.g., dog feces)* Fecal-contaminated food or waterDiagnosticsLaboratory tests: mild eosinophiliaSerology: positive ELISAImagingUltrasonographyCystic echinococcosis: unilocular, anechoic, smooth, well-defined hepatic cyst with or without daughter cystsEggshell calcifications within the wall of a hydatid cystCT scan: indicated for further evaluation of cystsAlveolar echinococcosis usually not well-defined, but shows infiltration of the liver and surrounding tissueTreatmentCystic echinococcosisMedical therapy: may be considered as the sole treatment for cysts < 5 cmDrug of choice: albendazoleUltrasonography/CT-guided percutaneous drainagedone in combination with medical therapy (albendazole)Surgery* Indications: > 10 cm, complicated cystsAny invasive procedure (drainage or surgery) of hydatid cysts should be performed with the utmost care to prevent spillage of cyst contents, which could cause life-threatening anaphylactic shock and/or secondary seeding of infection!

How well did you know this?
1
Not at all
2
3
4
5
Perfectly
44
Q

Incidental throid papilary microcarcinoma (o.5cm), after subtotal thyroidectomy for treatment of graves- what to do? * CT neckRepeat ultrasound in six monthsRight hemithyroidectomyTotal thyroidectomyRadioiodine ablation

A

Papillary carcinoma is the most Prevalent type of thyroid cancer, it features Palpable lymph nodes, and it has the best Prognosis compared to all other types of thyroid cancer. incidental thyroid carcinoma’ as it was detected unexpectedly in tissue resected for a benign pathology. Observational studies suggest that patients with these tumours do not need resection and can be kept under surveillance with repeat ultrasonography. In this case, the tumour has been resected as part of another procedure and certainly no further intervention is needed.With the increased use of various imaging tools, the incidence of “thyroid cancer” has increased dramatically over the last decade. Despite this increase in incidence, the mortality of the disease remains unchanged. This suggests that most of these newly diagnosed tumours are of little, if any, clinical significance.Learning PointsSmall, incidental thyroid tumours can be managed by surveillance.

How well did you know this?
1
Not at all
2
3
4
5
Perfectly
45
Q

increasing calf cramping on exertion, a history of smoking and hypertension, pale lower extremities, and absent pedal pulses, the combination of which suggests peripheral arterial disease (PAD).what is next treatment

A

Treatment of PAD Overview Intermittent claudicationFirst-line therapy [23][24]* Structured exercise therapyCardiovascular risk factor modificationCLI: Consider revascularization in addition to structured exercise therapy and cardiovascular risk factor modification.Risk modification [3][25]patients with PAD are at an increased risk of atherosclerotic cardiovascular disease (ASCVD) events such as MI or strokeSmoking cessationTreatment of dyslipidemiastatin therapy in all patients with PADGlycemic controlAntihypertensive treatmentSingle-agent antiplatelet therapy ; : aspirin OR clopidogrel (ticagrelor may also be considered) [3][7][28]Recommended in all patients with symptomatic PAD (reduces morbidity and mortality from cardiovascular events)Structured exercise therapy [3][22]Recommended first-line therapy for claudicationVasodilators [3][7][28]Preferred agent: cilostazol; a phosphodiesterase III (PDE3) inhibitorRevascularization [7][10][23]IndicationsLifestyle-limiting claudication despite optimal medical therapy and exerciseMAXIMIZE TABLETABLE QUIZRevascularization procedures for peripheral arterial disease Percutaneous transluminal angioplastyMay be combined with stent placement and/or atherectomyPeripheral artery bypass surgery: Open surgical bypass of the vascular stenosis with an autologous vein or prosthetic materialEndarterectomy (may be combined with endovascular treatment)Indications [10][33][34]Endovascular revascularizationConsider in the following situations:Short segment disease: stenosis < 10 cm or occlusion < 5 cmAortoiliac diseaseHigh-risk patients [10][33][34]Surgical revascularizationConsider in low- and average-risk patients with any of the following: [10][33][34]* Extensive and complex disease: long segment lesions (> 10 cm); multifocal lesionsLesions of the common femoral artery* Purely infrapopliteal disease* Chronic total occlusionUnsuccessful endovascular revascularization [10][33][3

How well did you know this?
1
Not at all
2
3
4
5
Perfectly
46
Q

Indications for bariatric surgery

A

Indications for bariatric surgery are based on a person’s BMI (body mass index), which needs to be 40 or greater to be considered or greater than 35 with a co-morbid conditions such as type 2 diabetes, hypertension, hyperlipidaemia, obstructive sleep apnea, non-alcoholic fatty liver disease, among others* must have failed non-operative weight loss attempts, * been cleared by a psychologist/psychiatrist and be able to give their informed consent.

How well did you know this?
1
Not at all
2
3
4
5
Perfectly
47
Q

Indirect vs Direct hernia difference

A

Indirect moves through the deep inguinal ring - Indirect in Inguinal ring. Direct moves through weakness of wall. - medial to inguinal ringFemoral- lateral and inferior to inguinal indirect inguinal hernia, where abdominal contents (typically bowel) pass through the inguinal ring and into the inguinal canal. The patient is experiencing abdominal pain and the hernia is irreducible, which is concerning for strangulation (although the swelling is not tender at this stage) and requires immediate surgical input.Femoral hernias are more common in women and of high risk of strangulation. However, these travel through the femoral ring. Direct inguinal hernias occur medial to the deep ring, where the aponeurosis of the abdominal wall is most weak.

How well did you know this?
1
Not at all
2
3
4
5
Perfectly
48
Q

Indirect vs Direct hernia difference

A

Indirect moves through the deep inguinal ring - Indirect in Inguinal ring. Direct moves through weakness of wall. - medial to inguinal ringFemoral- lateral and inferior to inguinal indirect inguinal hernia, where abdominal contents (typically bowel) pass through the inguinal ring and into the inguinal canal. The patient is experiencing abdominal pain and the hernia is irreducible, which is concerning for strangulation (although the swelling is not tender at this stage) and requires immediate surgical input. In this particular case it could be argued that the CT scan was unnecessary as the diagnosis of small bowel obstruction secondary to an incarcerated inguinal hernia should have been very clinically evident.Femoral hernias are more common in women and of high risk of strangulation. However, these travel through the femoral ring. Direct inguinal hernias occur medial to the deep ring, where the aponeurosis of the abdominal wall is most weak.

How well did you know this?
1
Not at all
2
3
4
5
Perfectly
49
Q

Intermittent soiling and perianal pain in an otherwise well individual.

A

Most likely an Anal fistula is an abnormal connection between the anus and the skin. This leads to a pathological opening from where leakage of stools can occur. The large majority of anal fistulae arise from anal crypts which have become infected (eg, perianal abscess). DDX for fistula , Whilst there are other conditions hat may be associated with anal fistula formation, most are a result of local sepsis. * Crohn’s disease, * Intestinal Schictosomiasis* actinomycosis, and * Chlamydia (eg, lymphogranuloma venereum) * anal fistula are radiation proctitis and malignancy. Crohn’s disease he might have concomitant malabsorption symptoms (eg, steatorrhoea, fat-soluble vitamin deficiency) and chlamydia may have associated painful inguinal lymphadenopathy and watery discharge from the urethra. Actinomycosis associated with anal fistula is an extremely rare finding.Intestinal schistosomiasis is associated with abdominal pain, bloody stools, diarrhoea, and strictures. Intestinal schistosomiasis is also a less likely cause of anal fistulae as compared to anal crypt infection.

How well did you know this?
1
Not at all
2
3
4
5
Perfectly
50
Q

Investigation of choice in SAH

A

This patient is presenting with classical features of subarachnoid haemorrhage (SAH), which include the hallmark ‘thunderclap headache’, nausea/vomiting, signs of meningeal irritation (photophobia and neck stiffness), focal neurological deficits, and loss of consciousness. The previous episode described likely represents a ‘sentinel bleed’ or ‘warning leak’, a minor haemorrhage that precedes 10-40% of major SAH. Hypertension and smoking are both risk factors.Urgent CT brain is the investigation of choice for suspected SAH, including a non-contrast scan to detect subarachnoid blood (which will appear hyperdense) and a CT angiogram to identify any potential culprit aneurysms. CT scanning is fast, widely available, and approaches 100% sensitivity in the first 3 days following SAH. In cases of high clinical suspicion but negative CT findings, lumbar puncture for xanthochromia and MRI can be used to further investigate. Formal cerebral angiography is used to either identify aneurysms when none are appreciated on non-invasive imaging, or to characterise detected aneurysms regarding their suitability for endovascular (coiling/stenting) versus open surgical (clipping) treatment.

How well did you know this?
1
Not at all
2
3
4
5
Perfectly
51
Q

Investigation/mx of choice of a groin mass?No cough impulse at mass, but signs of intestinal obstruction. What does this mean?

A

Irreducible/reducible only seen– cough refelxobstructed/stangulated –both signs obstruction, strangulated will show signs of local tenderness and systemically unwell (fever) A tender swelling in the groin of a patient with signs of intestinal obstruction supports the diagnosis of a strangulated hernia.—urgent surgical exploration with herniography This patient with a groin lump and clinical features of bowel obstruction (eg, nausea, vomiting, abdominal pain and distension, with tinkling bowl sounds: When the bowel is obstructed they become high pitched or tinkling as fluid drips from one distended and tympanic loop of bowel into another. ) most likely has an obstructed femoral hernia (older aage and female). Abdominal wall that is soft and distended- means no peritonism.Both obstructed and strangulated hernias tend to present with signs of bowel obstruction. A strangulated hernia may present with localised symptoms and signs (Richter hernia). In general, strangulated hernias are associated with toxic appearance and systemic symptoms (eg, fever). While most femoral hernias may show a positive cough reflex, this is only seen in reducible or irreducible femoral hernias. Obstructed and strangulated hernias do not generally show a positive cough reflex. Based on clinical suspicion of obstructed femoral hernia, this patient needs to undergo urgent surgical exploration of the groin with herniorrhaphy, no matter what the X-ray shows. Delaying the definitive surgical repair increases the risk of strangulation and necrosis of herniated bowel.

How well did you know this?
1
Not at all
2
3
4
5
Perfectly
52
Q

irregular superficial ulcer around his right medial malleolus. This has been present for about three weeks. There is moderate pain in the ulcer, which is relieved with elevation of the leg. On examination the area around the lower leg and ulcer is a dark purple rusty colour. His femoral pulses are of good volume and equal, and pedal pulses are strongly palpable. The ankle-brachial index in both legs is 1.0.

A

Compression bandages with moist wound dressingThe stem suggests a venous ulcer due to its location in the medial gaiter region, surrounding haemosiderin staining, and moderate pain that alleviates with elevation. The good arterial vascularity as evidenced by strong pedal pulses and normal ankle-brachial index reduces the likelihood of arterial malperfusion as the cause.Appropriate treatment at this stage would be compression bandages to reduce venous engorgement and moist wound dressing to control exudate and encourage epithelial regrowth. The presence of peripheral arterial disease would be a contraindication to direct compression - but that is not the case here.Skin grafting might be appropriate if the wound does not respond to simple measures. There is no need for any urgent vascular surgical opinion and nor is any other form of intervention (such as debridement) required as there are no apparent signs of infection or necrosis.Learning PointsA chronic ulcer around the ankle is most likely to be venous in origin.

How well did you know this?
1
Not at all
2
3
4
5
Perfectly
53
Q

IS VTE indicated?laproscopic Surg, female on OCP, no co morbidities.

A

Caprini thrombosis risk model which takes into account age, surgery type, medical history, and history of VTE as well as clinical evidence of peripheral venous disease. Interpretation of the Caprini model is as follows:Very low risk (<0.5%): 0 points- ambulation* Low risk (1.5%): 1-2 points- mechanical * Moderate risk (3%): 3-4 points- LMWH/mechanical High risk (>6%): >5 points - strong recommend VTE If risk were very low, then early ambulation would be recommended with no additional measures. If risk were low, then mechanical methods of prophylaxis would be indicated including intermittent pneumatic compression or graduated compression stockings. risk is moderate, a low molecular weight heparin such as enoxaparin given once per day until discharge would be appropriate. An alternative option may be graded compression stockings to be worn until discharge.If her risk were high, then pharmacological prophylaxis is strongly indicated.VTE prophylaxis is usually continued until the patient is fully ambulatory or until discharge. Although high-risk patients or those with prolonged immobility on discharge may be prescribed a longer duration (10-14 days).Following VTE risk, an assessment of bleeding risk should occur. EGVTE RISK: This patient is at moderate risk for a venous thromboembolic complication, in that she has two risk factors:Abdominal surgery (2 points)Oral contraceptive pill (1 point)BLEEDING RISK In this patient, undergoing laparoscopic surgery with no medical history, her bleeding risk is low. Therefore prophylactic anticoagulation is indicated.

How well did you know this?
1
Not at all
2
3
4
5
Perfectly
54
Q

Knee arthroscopy- when used

A

Knee arthroscopy is a common surgical procedure that is more frequently performed for meniscal tears, removal of loose bodies, focal cartilage lesions, or ligament reconstructions. It has a limited role in the management of an arthritic knee.

How well did you know this?
1
Not at all
2
3
4
5
Perfectly
55
Q

leave alone “groin” discomfort

A

Groin ‘hernias’ that are not clinically evident and only found on imaging are unlikely to be of any clinical significance.The pain that the patient experienced is almost certainly musculoskeletal in origin, especially with the finding of the area of tenderness. With a normal physical examination that does not find any clinically apparent inguinal hernia, all the patient needs is reassurance and the explanation that his pain will settle on its own accord. He should be advised that further heavy lifting or straining might aggravate things. Whilst an ultrasound examination is a very commonly performed examination in the assessment of patients with groin pain, the report in this case can be misleading. The deep inguinal ring has a normal diameter of 2 cm, so the report of a ‘ 1 cm defect’ is of no clinical significance. Likewise herniation of some fat into the canal is very unlikely to be of clinical relevance. Unfortunately this scenario is all to common, with these patients being referred for a surgical opinion and consideration of a ‘hernia repair.’ ”. Even if this patient was found to have a hernia, recent studies have shown that small, minimally symptomatic, first occurrence hernias do not necessarily require repair; instead, they suggest counselling the patient on red-flag symptoms suggestive of incarceration and strangulation, with advice to seek prompt evaluation in these instances.

56
Q

Left iliac fossa pain. Temp 37.4C, blood pressure 135/90 mmHg and pulse 100/min. PE: There is some tenderness in the left iliac fossa and the rest of the physical examination is normal. A CT confirms the clinical diagnosis of acute diverticulitis. There is a 5 cm length of the sigmoid colon affected, with some pericolic fat stranding and thickening of the bowel wall. No other abnormalities are detected.…two-day history of constant lower abdominal pain associated with loose stools. Her medical history includes hypertension, gout, type 2 diabetes, and a similar episode of abdominal pain two years previously. She looks well, with a blood pressure of 130/90 mmHg, pulse 90/min, and temperature 37.4C. abdomen is soft, with some tenderness in the left iliac fossa. Her haematological and biochemical values are all within normal limits. HbA1c at her last medical checkup one month ago was 68.3 mmol/mol. A CT scan is ordered and shows thickening of the sigmoid colon and no other abnormalities.

A

Mx of acute diverticulitis: Complicated vs uncomplicated (simple) Simple diverticulitis- no local complications ( perforation, abscess, fistula)Some 75% of cases of diverticulitis are uncomplicated. Uncomplicated can be managed as an outpatient with review, Analgesia.Antibiotics would be appropriate for high-risk patients who could be defined as:* immunocompromised* high C-reactive protein assays (>140 mg/L)* high white blood cell count (>15)* CT evidence of long segment involvement (>6.5 cm)* local complication (fluid collection, abscess, perforation)Complicated Diverticulitis- long segement 6.5 cm or more of gut In patient -Significant co-morbidities (including immunosuppression, chronic steroid use, uncontrolled diabetes mellitus)Evidence of infectious parameters (fever >39C, white cell count >15 x 106/L)Inability to tolerate oral intakeLimited home supportCT evidence of pneumoperitoneum, abscess formationPatients over the age of 65 are more likely to be managed in a hospital, but this is not an absolute indication. Eg 2: in this case, the patient has diabetes and this co-morbidity is a stronger reason than any of the other options to consider the need for in-patient management of her acute diverticulitis.Learning PointsUncomplicated diverticulitis can be managed in an out-patient setting - except where various co-morbidities, such as diabetes, indicate the need for hospitalisation. colonoscopy this should be left until the acute inflammatory episode has resolved.

57
Q

Mastalgia- MX

A

Clinical featuresTender, firm, swollen, erythematous breast (generally unilateral)Pain during breastfeeding* Reduced milk secretionFlu-like symptoms, malaise, fever, and chillsIn some cases, reactive lymphadenopathyDiagnosticsClinical diagnosisBreast milk cultures or imaging may be required if there is no response to initial treatment. TreatmentIn nursing mothers, breastfeeding with alternate breasts is recommended every 2–3 hours.Analgesics (e.g., ibuprofen)* Cold compressesAntibiotic treatmentOral penicillinase-resistant penicillin or cephalosporin (e.g., dicloxacillin or cephalexin)In the case of methicillin-resistant Staphylococcus aureus (MRSA): clindamycin or trimethoprim-sulfamethoxazole (TMP-SMX) or vancomycin for severe cases In the case of inadequate response to initial treatment:Initiate treatment according to breast milk culture results.Consider an underlying breast abscess, which requires surgical drainage.

58
Q

Mechanical prophylaxis methods include:• Intermittent pneumatic compression (IPC) or sequential compression device (SCD) including foot impulse devices• Graduated compression stockings (GCS)

A

IPC includes sleeves/cuffs applied to legs and garmentswrapped around the foot (foot impulse devices) connected to an air pump that sequentiallyinflates to improve venous circulation in the limbs. IPC is more effective than GCS in preventing DVT in surgical patients. IPC is recommended over GCS in patients withmoderate to high VTE risk that are not receiving pharmacological prophylaxis or in patients at very high VTE risk that are on combined mechanical and pharmacological prophylaxis. GCS apply pressure on the leg muscles to squeeze the vein valves starting with the greatest degree of compression at the ankle and decreasing the level of compression up the leg. This helps ensure that blood flows upwards to the heart instead of refluxing downwards

59
Q

most likely causative organism of wound infection

A

inflammation around some local abscess formation in the wound. Escherichia coli and Bacteroides sp (usually fragilis) are the two most common causes of post-operative wound infection, particularly when some part of the gastrointestinal tract has been opened. Of all the organisms mentions, Escherichia coli is the most likely cause of the wound infection. Salmonella sp do not cause wound infections. Streptococcus epidermidis is a common cause of cellulitis rather than wound infection per se. Whilst Staphylococcal infections are a common cause of dermal and subnormal abscesses, in these circumstances the infection is most likely to be due to a gut-derived organism. Clostridium perfringens is a relatively common cause of food poisoning and when it does cause tissue infection, it usually leads to necrosis.Learning PointsEscherichia coli and Bacteroides sp (usually fragilis- grm - anaerobes) are the two most common causes of post-operative wound infection, particularly when some part of the gastrointestinal tract has been opened.

60
Q

most likely cause of pancreatitis-serum lipase of 220 U/L (0-60).

A

The two most common causes of acute pancreatitis are gallstones and alcohol. Gallstones are the most common cause of acute pancreatitis, being responsible for at least 40% of cases. Chronic alcohol consumption is the next most common cause, accounting for about 35% of cases. The third commonest cause of acute pancreatitis is iatrogenic (post ERCP or cardiopulmonary bypass), adding up to about 20% of all cases. The other recognised causes (eg hypertriglyceridaemia, hypercalcalcaemia) all total less than 5% of cases. Whilst some drugs can be associated with acute pancreatitis, the NSAIDs have yet to be incriminated. This patient has known gallstones and even though he might have enjoyed a few drinks the evening before his presentation, it would have been the documented gallstones that would have been likely trigger for this episode of acute pancreatitis.

61
Q

motility disorder, how is swollowing affected

A

motility disorder as the dysphagia initially worse with fluids rather than solids.

62
Q

Mx of head of femur fracture , classification and mx of each class

A

Pipkin Classification Pipkin I Luxational fracture of the femoral head: The fracture line lies below the fovea capitis and not in the weight-bearing portion (horizontal fracture).Surgical: open reduction internal fixation, e.g., with a traction boltConservative: immobilizationPipkin II Luxational fracture of the femoral head: The fracture line lies above the fovea capitis. The fragment is attached to the femoral head ligament and thus in the weight-bearing portion (vertical fracture).Surgical: open reduction internal fixation, e.g., with a traction boltPipkin III Pipkin I or II fracture in combination with a medial fracture of the femoral neckSurgicalYoung patients: open reduction internal fixation with maintenance of the femoral headElderly patients or those with predispositions: total hip replacementPipkin IV Pipkin I or II fracture in combination with an acetabular fractureSurgicalYoung patients: open reduction internal fixation with maintenance of the femoral headElderly patients or those with predispositions or instabilities: total hip replacement

63
Q

mx of subcapital (femor neck fracture), what classification is used, and what surgery does each state require.

A

The main forms of intracapsular fracture management include -internal fixation (either multiple cancerous sores or single sliding hip screw), -or arthroplasty (either hemi or total replacement). The choice depends on the degree of displacement (Garden stage) and comorbidities of the patient. -Garden state I/II are managed with internal fixation regardless of age. -Generally, with patients under 65 it is best to aim for internal fixation as the arthroplasty may not last the patient’s life expectancy and there is greater risk of re-intervention. -Garden III/IV and older than 65 are usually managed with either hemi or total hip arthroplasty. -The decision between hemiarthroplasty and total is controversial and can be very patient dependant. Some factors that may push the clinician towards using a total hip replacement include good pre-fracture mobility, and no cognitive impairment. This patient is 75 and has suffered a valgus impacted subcapital fracture. ( see image) This is best managed with cannulated hip screws or dynamic hip screws. Her pre-existing cognitive impairment as well as mobility issues make her less of a candidate for arthroplasty but the principal deciding factor is the type of fracture.The dynamic hip screw or sliding hip screw fixation is designed for fixation of certain types of proximal femur or hip fractures. The screw is a large cancellous lag screw that can glide freely in a metal sleeve. The sleeve is attached to a plate which is fixed to the lateral femoral cortex with screws.

64
Q

Mx vertebral compression fracture in elderly

A

Simple analgesia and mobility as tolerated - asap to encourage core musculature focal pain in her lower back from a sudden onset of lifting a heavy item. This is indicative of mechanical trauma, such as a vertebral compression fracture. Typically, compression fractures are fragility fractures common in patients with a background of osteoporosis - a common condition associated with the elderly population. This occurs when bone cortices are thinned and/or cancellous bone has decreased trabecular continuity. Minimal trauma would result in a compression or wedge fracture between each vertebral.The majority of patients can be treated with observation and adequate pain relief with the gradual return of activity/mobilisation. Physical activity should be resumed as quickly as possible if tolerated. This is useful for gait and core strengthening when the patient can tolerate the level of activity.Indications for and timing of surgical management (e.g. vertebral augmentation) remains controversial. There is a limited beneficial effect in vertebral augmentation, and most patients can be treated successfully with conservative management. Surgical management may benefit specific patient groups - debilitating pain and in those who are not improving with conservative management.

65
Q

Name key myotome deficits in cervical radiculopathy (C5-T1)

A

C5 -elbow flexionC6- wrist extensionC7- wrist flexion, fingers extensionC8 - finger flexionT1- finger abduction

66
Q

navicular and base of 5th meta tarsal

A

mid foot Ottawa rules- point tenderness in two points and inability to WB ( 4 steps) Note- the base of the 5th metatarsal rests against the cuboid- then the tuberosity therafter. up until age 12-14- apophysis (growth plate on tuberosity)

67
Q

obvious effusion around his left knee. This is not reddened, tender or unduly warm. There is some varus deformity and extension/flexion is limited by pain. There is crepitus upon movement with no joint line tenderness.A plain X-ray confirms effusion, with narrowing of the joint space and osteophyte formation medially. An aspirate of clear yellow fluid is obtained, which has a white cell count of 10.0x109/L (4.0 - 11.0).Diagnosis? Treatment

A

medial compartment osteoarthritis. This is supported by the chronic progression of the disease, clinical findings of limited flexion/extension, and presence of osteophytes on X-ray.Nonoperative measures which has proven to be beneficial includes simple analgesia (paracetamol and PRN NSAIDs), weight loss if indicated, and low impact exercises. A thorough non-operative program should be initiated prior to any surgical intervention.Learning PointsFirst line management of osteoarthritis includes simple analgesia, weight loss if indicated, and low impact exercise.NOTE-Tibial osteotomy is an available surgical option for treating varus alignment due to medial osteoarthritis, however non-operative measure should be initiated first.Platelet-rich plasma injection (PRP) is another option for symptomatic management of osteoarthritis. However most clinical guidelines do not include PRP as part of their treatment plan as it has limited evidence on its efficiency to improve patients’ symptoms

68
Q

Ottawa anke rules- Midfoot and ankle

A

There are two components, assessing for ankle and midfoot fractures. A patient with traumatic ankle pain qualifies for ankle radiographs if they have any of the following:point tenderness at posterior edge (of distal 6 cm) or tip lateral malleoluspoint tenderness at posterior edge (of distal 6 cm) or tip medial malleolusinability to weight bear (four steps) immediately after the injury and in emergency departmentA patient with traumatic midfoot pain qualifies for foot radiographs if they have any of the following :point tenderness at the base of the fifth metatarsalpoint tenderness at the navicularinability to weight bear (four steps) immediately after the injury and in emergency department

69
Q

Intermittent Claudication (PAD) - name the ABI categories and the Inx for each

A

PAD- use ABI to measure- measure of arterial flow. SBP of dorsalis peadis/ SBP of brachial ABI:* > 1.4 Medial calcific sclerosis (ass with DM and CKD) with noncompressible vascular wall* 1.3-09- normal * less than 0.9 is slight restriction of flow – * mild-moderate 0.5-0.9 – investigate with duplex ultrasound to find stenosis* severe- less than 0.4– will need imaging (angiography/ MRI) as a prelude to some form of surgical intervention. Choice of investigation- Imaging [7][18][19]Angiography: preferred modality for assessment for revascularizationModalities [18]First-line: MR angiography (MRA) without and with IV contrast Contraindications for MRI (e.g., prior stent placement): CT angiography with IV contrast (CTA)Digital subtraction angiography: considered the gold standard for PAD; uncommonly used [19]* Findings: demonstration of site(s) and extent of arterial occlusion or stenosis and collateral blood flowMRA has a lower risk of nephrotoxcicity than CTA, and also better visualises strongly calcified vessels , making it more suitable in renal impairment and diabetesDSA- was gold standard, but invasive and high contrast load.

70
Q

Painful arc at specific points: 60-120, 120-180

A

Painful arc testProcedure: The examiner instructs the patient to abduct and raise the extended arm.Findings and significance: * Negative test (physiological): The patient’s arm can be repeatedly raised and lowered between 0–180°.Positive test: painful arc* Pain between 60–120° indicates subacromial impingement * Pain from abduction/elevation at 120–170° indicates a pathology of the AC joint * Pain during the entire movement (0°–180°) nonspecific indication of a glenohumeral pathology (e.g., osteoarthritis or frozen shoulder)https://youtu.be/zMnjj7YV-40Impingement syndrome can make an assessment of the actual range of motion in the shoulder difficult due to pain. To nullify the effects of, e.g., subacromial impingement, abduction in external rotation can be performed. As a result of this, the space between the acromion and the supraspinatus tendon is enlarged, leading to reduced pain.

71
Q

painless swelling on the right side of his face. slight droop at the right corner of his mouth and there is a 4 cm diameter, well circumscribed swelling immediately anterior to the right tragus.lesion is firm and mobile, with no apparent tethering to the overlying skin. Pleomrphic adenoma or Parotoid adenocarcinoma

A

Learning PointsFacial nerve weakness in the presence of a swelling in the parotid region indicates malignancy until proven otherwiseThe parotid space contains both the parotid gland and external carotid artery, as well as the intra and extraparotid lymph nodes, retromandibular veins and extracranial branches of the facial nerve.The majority of pathologies related to the parotid space arise from the parotid gland, and may be benign or neoplastic in nature. pleomorphic adenoma- benignadenocarcinoma- malignantPhysical examination of these pathologies is often non-specific, however the facial weakness indicates invasion of the facial nerve, a strong indicator for an underlying malignant process. The facial nerve passes through the superficial and deep part of the parotid gland and facial nerve weakness in the presence of a parotid swelling suggests malignant infiltration and a parotid adenocarcinoma.Metastatic deposits do occur within the parotid gland, however are significantly less common than primary tumours of the parotid gland.

72
Q

Pateint px with runny nose- triggers of cold air and exertion

A

chronic non-allergic rhinitis (formerly called vasomotor rhinitis; renamed due to lack of evidence of vascular aetiology). His triggers of cold air and exertion suggest the most common subtype, termed non-allergic rhinopathy.In addition to trigger avoidance, management is symptom-directed:For patients with predominant rhinorrhoea (as in this case), intranasal ipratropium is first-line. It has a local action on the cells onto which it is sprayed and works by inhibiting the secretion of mucus, with onset within 15 minutes. If symptoms persist, intranasal corticosteroids or antihistamines can be added.For patients with predominant congestion, intranasal corticosteroids and/or antihistamines are first-line, with oral decongestants added if symptoms persist. If symptoms persist despite the above, allergy testing should be considered.

73
Q

patellar tendonitis vs patellar bursitis

A

Patellar tendonitis refers to inflammation of the patellar tendon. Also called “jumper’s knee”, it is commonly seen in athletes, particularly those who play sports such as volleyball and basketball. In patellar tendonitis, the pain typically gets worse with repetitive movement and activity, rather than relieved . In addition, most patients will have localised tenderness over the patella tendon. if no focal tenderness, making patellar tendonitis a less likely diagnosis. Bursitis refers to inflammation of a pre-patellar bursa, the fluid filled sac superficial to the knee joint itself. Bursitis in the knee would likely present with a higher degree of swelling, along with erythema.

74
Q

patient is manifesting neurological deterioration in the days following aneurysmal subarachnoid haemorrhage (SAH)

A

Vasospasm presents with focal and/or global neurological deterioration in the days following SAHDelayed cerebral ischaemia is caused by vasospasm, believed to be triggered by blood products in the subarachnoid space irritating the abluminal surface of blood vessels with subsequent ischaemia in the corresponding cerebral tissue. Symptomatic vasospasm occurs in ~30% of SAH patients, and this falls within the peak risk window of 3 - 14 days post SAH. Classic clinical features include confusion and/or increased sedation, plus or minus focal phenomena including territorial motor deficits or cranial nerve palsies.in scenario: The nurse calls you to see him and you find him drowsy and disoriented, complaining of new headache, and you notice a left-sided facial droop and 4/5 weakness of the left upper limb. He is haemodynamically stable, afebrile

75
Q

patient with med Hx of Mysthenia gravis. Now with acute cholecystitisTreatment?

A

Acute cholecystitis is empirically treated with amoxicillin/gentimicin/metronidazole - gram + (enterococcus: in GUT) and grm - (enterobacteria: BT) coverage BUT gentamycin is CONTRAINDICATED IN MG- causes prolonged muscle relaxation Acute cholecystitis is usually caused by Enterobacteriaceae (Gram-negative - KEEPS) and Enterococcus faecalis(Gram positive: Enterococcus faecalis is the most common human pathogen, but Enterococcus faecium has become increasingly prevalent in hospital-acquired infections). Amoxicillin and gentamicin cover both these Gram-negative and Gram-positive bacteria. Metronidazole has activity against anaerobic bacterial infections which constitutes the majority of intestinal flora.However, in this case, Piperacillin/Tazobactam is the correct answer as the patient has a history of myasthenia gravis which is a contraindication to using gentamicin due to the risk of prolonged neuromuscular blockade. Hence, an alternative antibiotic regimen will be required.

76
Q

patient’s mobility will be reduced for a significant period post-operatively, thromboprophylaxis is important. Chemical vs mechanical VTE prophylaxis

A

Two factors point away from chemical prophylaxis in this scenario: * the patient has baseline low risk for VTE by virtue of age and lack of comorbidities, * and increased risk of bleeding is especially concerning after neurosurgery as intracranial haemorrhage carries a poor prognosis. Intermittent pneumatic compression provides greater protection when the patient is immobile than graduated compression stockings, and are therefore the appropriate choice in this scenario.Learning PointsLocal compression devices alone can be used for the young, fit patient undergoing a minor procedure.Chemical prophylaxis must be weighed against the risk of bleeding in a high-risk procedure.

77
Q

Pattela Bursitis vs Osteochondroma. Is osteochondroma benign or malignant

A

Patella bursitis is common in patients who kneels often. Symptoms may include generalised pain and swelling in the knee, which can be red or warm to touch.Osteochondroma of the proximal tibia is a common benign primary bone tumour. It is usually asymptomatic but can be painful and palpable near the ends of the long bones. X-ray findings include pedunculated or sessile lesions

78
Q

Pes Ansures Bursitis?

A
79
Q

Pivoted and experienced a sensation of his knee giving way. He fell, but was able to get up and hobble off the field. Over the next few hours he noticed some swelling around the knee. On examination, the knee is swollen, the bulge sign is positive, with no clear bony deformity. There is tenderness along the medial joint line. He is unable to fully extend the knee. There is no laxity on anterior or posterior movement of the tibia against the femur.

A

Bulge sign positive- miniscal tear Medial joint line tenderness- Within jointant/post draw test negative- not cruciate ligamentsinability to fully extend- catching– medial meniscus buckle handle characteristic of a meniscal injury, the findings of an inability to fully extend the knee supports the diagnosis of a bucket handle medial meniscus tear where the meniscal fragment can displace into the intercondylar notch and act as a physical block to full extension. Whist similar types of non-contact pivoting injuries can lead to cruciate tears, these types of injury are usually associated with a rapid swelling about the knee. In addition, there would typically be joint laxity and anterior or posterior displacement of the tibia.Learning PointsA medial meniscal tear is associated with an inability to fully extend the knee.

80
Q

Postoperative confusion is a common sequelae after surgical events, particularly major procedures in elderly patients. On examination her blood pressure is 130/94 mmHg, pulse 100/min, temperature 37.2C and oxygen saturation 96% on room air. The rest of the examination is unremarkable and her abdomen appears to be soft and the surgical sites intact and healthy-looking. Her haematological and biochemical screens earlier that day showed normal results. A chest X-ray and an ECG are both normal,, excluded… now?

A

means of countering the confusion can be considered. Taking a patient out of their own familiar environment and routine can be disturbing for them and nocturnal confusion is common. Simple measures such as providing human contact and lighting the room may be all that is necessary to restore calm. If that does not work, some form of sedative or hypnotic may be required.Learning PointsSimple measures, such as keeping the patient in a well-lit room, will manage most cases of post-operative confusion.

81
Q

pregnant ( 27 weeks) with acute cholecyctitis ( prolonged RUQ pain, low grade fever, RUQ tendernesss)

A

In most instances it will settle of its own accord, with gut rest and intravenous fluid replacement. During pregnancy, women are at increased risk of gallbladder disease due to increased gallbladder stasis and production of bile with decreased chenodeoxycholic acid and increased cholesterol. This woman is presenting with an episode of cholecystitis, suggested by her prolonged right upper quadrant pain, low-grade fever and right upper quadrant tenderness. She also has a history suggestive of two prior episodes of biliary colic. Laparoscopic cholecystectomy should be considered for all pregnant women with symptomatic gallstones.Evidence has shown that conservative management compared with operative management is linked with significantly higher maternal-fetal complications, including preterm birth and low fetal growth. Patients in the first, second and third trimester who are managed nonoperatively will experience recurrent symptoms secondary to cholelithiasis in 92%, 64% and 44% of cases respectively. Additionally, 23% of patients managed nonoperatively will develop acute cholecystitis or gallstone pancreatitis, which is associated with a high rate of fetal loss. Operative management is therefore recommended, and a laparoscopic cholecystectomy can be safely performed during any trimester.

82
Q

Presentation suggestive of a CSF leak

A

The most common presentation of a patient with a cerebrospinal fluid leak is clear rhinorrhoea that may be accompanied by a headache, described as ‘pressure-like’ that may be positional in nature. eg as the headache worsens throughout the day after changing position from supine to standing. Other supportive factors are the preceding event being traumatic, craniofacial trauma eg with a deviated nasal septum being evidence of this trauma. CSF leaks also present with clear rhinorrhoea, which is also present in this patient. The most common aetiology is structural compromise due to craniofacial trauma.A CSF leak can occur when there is disruption of the meninges, leading to an open communication between the subarachnoid space and other spaces.

83
Q

Prick finger gardening, diffusely swollen, held in a flexed position.

A

Flexor tenosynovitis is a closed space infection of the flexor tendons and synovial sheaths. The four cardinal Kanavel’s signs of flexor tenosynovitis are: (1) swelling over the affected tendon, (2) digit is held in partial flexion at rest, (3) tenderness over the affected tendon (whole tendon on flexor/extensor surface) and (4) pain along the tendon sheath on passive digit extension.Prompt diagnosis is required to prevent tendon necrosis and deeper spread of infection.Pyogenic flexor tenosynovitis is likely secondary to the bacteria being introduced from her prick on the rose thorn. Subcutaneous abscess will not have tenderness along the entire tendon.Cellulitis will present poorly defined erythema and swelling, without the flexor tendon findings. Felon is a subcutaneous infection of the distal fingertip pulp, a closed sac of connective tissue distal to the distal interphalangeal joint. Flexor tendon sheath haematoma is less common, particularly with history of a garden prick from a rose thorn which is likely to be the source of introducing an infection.Learning PointsFlexor tenosynovitis should be suspected in patients with fusiform digit swelling, held in partial flexion at rest and associated with significant pain on passive extension and tenderness on palpation the tendon.

84
Q

psuedo obstruction vs post surgery paralytic illues mx

A

psuedoobstruction- occurs in the elderly, after surgery (especially ortho,cardiac and vertebral), electrolyetsMx - In most instances it will settle of its own accord, with gut rest and intravenous fluid replacement.pharmacological: DOC- neostigmine (erythromycin-prokinetic: Metoclopramide:A dopamine-2 receptor and serotonin receptor antagonist. Commonly used as an antiemetic and to promote gastrointestinal motility in patients with gastroparesis. Adverse effects include extrapyramidal symptoms (e.g., dystonia), likely secondary to dopamine antagonism.if ceacum massively distended (>10cm) -decompression (colposcopy) If there is massive colonic distension, with the caecum measuring more than 10 cm or there is evidence of peritonism, then some form of active intervention is required. Decompression can often be achieved by colonoscopy and placement of a decompression tube. pseudo-obstruction. This is a well-recognised, but uncommon event, most often occurring in the elderly population. It has many different precipitating causes and typically occurs after pelvic or spinal surgery and can be a sequela of other orthopaedic surgery. It can also occur after trauma, cardiac events or with electrolyte imbalance. In most instances it will settle of its own accord, with gut rest and intravenous fluid replacement. If there is massive colonic distension, with the caecum measuring more than 10 cm or there is evidence of peritonism, then some form of active intervention is required. Decompression can often be achieved by colonoscopy and placement of a decompression tube.In instances where colonoscopy is not thought to be appropriate, pharmacological measures with a cholinesterase inhibitor (neostigmine) or a prokinetic agent may be used. Prokinetics such as erythromycin or metaclopramide are not as effective as neostigmine. Lactulose is contraindicated as this may worsen the situation by encouraging bacterial fermentation. Alvimopan (a peripherally acting opioid receptor antagonist) can be effective in the management of post-operative ileus, but its place in the management of pseudo-obstruction has yet to be shown. Medical therapies have less risk than mechanical instrumentation, but may be less effective.Learning PointsNeostigmine can be effective in the treatment of pseudo-obstruction which is not resolving of its own accord.

85
Q

Radiological signs of osteoarthritis

A

SCiSSor* Subchonsral cystsIrregular joint space narRowingSubchondral sclerosisOsteophytesosteoarthritis, plain X-rays are the most appropriate initial investigation; they are easily accessible and provide enough information to make the diagnosis. Information to be sought on the X-ray to support the diagnosis of osteoarthritis would include:Loss joint spaceOsteophyte formationSubarticular sclerosisSubchondral cystsThis can be remembered with the mnemonic ‘LOSS’.

86
Q

Shoulder pain- shortly after “over head activity”* no apparent deformity or swelling in the shoulder region. There is tenderness over the subacromial bursa and limitation of abduction at the shoulder joint. The patient is unable to raise his arm above his head. External rotation is also limited.

A

Rotator cuff tearTriggers- risk factors for rotator cuff injuries include age >40 years, Overhead activities, and repetitive rotator cuff strain (e.g. repeated overhead movements over a long period).Rotator cuff injury most commonly presents with painful abduction and external rotation of the shoulder

87
Q

signs of billary duct pathology but hx of gastric bypass surgery and cholesysectomy.

A

The clinical picture fits for biliary pain and the abnormal liver function tests support this (raised billi and alp). She has a dilated common bile duct and now needs more detailed imaging of her biliary systemThis means that there is no easy way of passing an endoscope to her ampulla of Vater. Of the various imaging modalities provided, MRCP will be the least inconvenient and should give adequate definition of the biliary tree. She may well have a stone in her common bile duct. If a stone is found, then some form of laparoscopically-assisted ERCP may provide the solution to the problem.Learning PointsA Roux-en-Y gastric bypass alters the anatomy such that endoscopic access to the biliary tree can be very difficult.

88
Q

Spontaneous, unilateral nipple - green in colour. 48 y/o women

A

discharge suggests a pathological cause. Whilst duct papilloma is the commonest cause of a pathological nipple discharge, the material is usually clear or bloody. The discharge in duct carcinoma in-situ is often either clear or bloody. In this case, the discharge is a greenish colour and the underlying cause is more likely to be duct ectasia. Mammary duct ectasia is the most common cause of greenish nipple discharge. Look at age of women The absence of breast tenderness or swelling makes mastitis less likely. Paget’s disease of the nipple is rare and typically features visible nipple changes such as scaling or retraction and not discharge. Regardless, tissue diagnosis is important for all pathological discharge to exclude carcinoma.Duct Ectasia- congealed/thickened luminal secretion, stasis leading to periductal inflammation and fibrous obliteration. Epidemiology: Most common in perimenopausal women Peak incidence: 40–50 years Risk factors: tobacco use, congenital nipple malformations Clinical features: Unilateral, sticky, multicolored discharge (e.g., gray, greenish or bloody discharge)* Nipple inversion* Firm, stable, painful mass under the nipple (may mimic breast cancer)May progress to a breast abscess Diagnostics: * Mammography and/or ultrasound: can be used to determine mammary duct diameter Biopsy (if any suspicious or inconclusive imaging findings) . TreatmentUsually not necessary (most cases resolve spontaneously)Antibiotic therapy if infectedSurgical excision for persistent lesions

89
Q

Surgical technique for each type of femur injury

A

Definitive management is surgical (Table 1), however the specific procedures depending on the type of fracture sustained, amongst several other factorsFracture TypeSurgical OptionSummaryDisplaced subcapitalHip Hemiarthroplasty*Replacement of the femoral head and neck via a femoral component fixed in the proximal femurInter-trochanteric and Basocervical*Dynamic Hip Screw (or short IM nail)Consists of a lag screw into the neck, a sideplate, and bicortical screws. The lag screw is able to slide through the sideplate, allowing for compression and primary healing of the boneNon-displaced intra-capsularCannulated hip screws**Three parallel screws in an inverted triangle formationSub-trochantericAnterograde Intramedullary Femoral NailThe titanium rod is placed through the medullary cavity of the femur for stabilisation Table 2 – Summary of Surgical Options for NOF Fractures; *consider total hip arthroplasty in systemically well patient who was living independently prior to injury; **can also consider hip hemiarthoplasty or total hip arthroplasty

90
Q

ten-day history of passing frothy, cloudy urine. Upon further questioning, she also describes passing streams of urine interrupted by “bubbling”. She does not have any other current health issues other than an episode of lower abdominal pain two months ago which lasted about three days and then settled of its own accord. Her medical history includes a hysterectomy when she was aged 40 and a pyelolithotomy when she was aged 68.

A

Learning PointsPneumaturia is best investigated with a CT abdomen.This patient is describing pneumaturia indicated by symptoms of frothy, cloudy urine, interrupted urine stream and bubbling. Gas within the bladder is suggestive of a fistula and further direct visualisation will be required to determine the cause.Of the various potential causes of gas in the urine, the commonest is an enterovesical fistula. Of the many possible causes of fistulous communication between the gut and the bladder, the commonest is diverticular disease. This would be the most likely explanation in this case, particularly as the patient had an episode of undiagnosed pain preceding these symptoms, which could have been diverticulitis. The best investigation for pneumaturia and suspected enterovesical fistula is a CT abdomen. The CT is likely to identify any mass lesion adjacent to the bladder (eg diverticular or tumour mass).CT urogram is a useful investigation for visualisation of the urinary tract including collecting systems, ureters and bladder. This is especially helpful in diagnosing cancers of the bladder but not fistulas. (filling defects)Colonoscopy may show a cause of enterovesical fistula such as a diverticular disease but is less useful in assessing the full extent of the disease and path of the fistula.Cystoscopy allows direct visualisation of the bladder and biopsy of a lesion but does not identify extraluminal content.Pelvic ultrasounds do not provide detailed anatomical information regarding the complexity of a fistula.

91
Q

tenderness of palpation at the medial tuberosity of the calcaneus, dorsiflexion of the toes and foot increases with palpation. There is limited ankle dorsiflexion

A

plantar fasciitis – an inflammation of the aponeurosis of its origin at the calcaneus. Risk factors include obesity, weight-bearing endurance exercise i.e. running and dancing. Plantar fasciitis presents as insidious sharp heel pain, often when getting out of bed in the morning. Patients may prefer walking on their toes initially, and the pain gets worse at the end of the day after prolonged standing. Physical exam suggests tenderness of palpation at the medial tuberosity of the calcaneus, dorsiflexion of the toes and foot increases with palpation. There is limited ankle dorsiflexion due to tight Achilles tendon.The first-line management is primarily pain analgesia with physical therapy which involves stretching and self-massage. Heel insoles can also be used to provide support and relieve tension as the heel is raised.Extracorporeal shockwave therapy (ESWT) for plantar fasciitis would be second-line management. Surgical interventions are rarely performed and only indicated if conservative management has failed.Steroid injections should very rarely be used for the increased risk of plantar fascia rupture.

92
Q

Tests for cruciate ligamentsTests for collateral ligaments

A

Tests for cruciate ligamentsAnterior drawer test and posterior drawer testProcedureThe examinee lies supine with hips flexed to 45° and knees flexed to 90°.The examiner sits on the examinee’s feet to secure them in place, holds the leg at the upper ⅓ of calf with both hands, and pushes the leg forward (anterior drawer test) or backward (posterior drawer test).InterpretationLaxity at the endpoint represents a positive test.A positive anterior drawer test indicates anterior cruciate ligament tear.A positive posterior drawer test indicates posterior cruciate ligament tear.Lachman testProcedureThe examinee lies supine and flexes the knee 20–30° with the heel of the foot resting on the examining table.The examiner stabilizes the femur and moves the proximal tibia anteriorly.Interpretation: Excessive anterior movement indicates anterior cruciate ligament tear. Tests for collateral ligamentsValgus stress test and varus stress testDescriptionThe examinee lies supine with the knee flexed to 30°.The examiner holds the examinee’s ankle with one hand and uses the other hand to apply pressure to the medial (valgus stress test) or lateral (varus stress test) aspects of the knee joint.InterpretationLaxity at the endpoint is interpreted as a positive test.A positive valgus stress test indicates medial collateral ligament injury.A positive varus stress test indicates lateral collateral ligament injury.

93
Q

Tests for a knee effusion

A

Tests for joint effusionPatellar tap testProcedureThe examinee lies supine with the knee extended.The examiner pushes the fluid from the suprapatellar pouch to the knee joint with one hand and uses the other hand to gently tap the area over the patella.Interpretation: floating of patella indicates joint effusionKnee bulge testDescription: The examiner applies pressure above the patella to milk fluid from the suprapatellar pouch and then applies pressure behind the lateral margin of the patella.Interpretation: The appearance of a bulge on the medial aspect of the knee indicates the presence of knee joint effusion.

94
Q

The symptoms and physical findings - reddened hemiscrotum with pain over 1-2 days, tenderness and fever.

A

The symptoms and physical findings fit for epididymitis - The commonest cause is reflux of infected urine - usually urinary retention in older men. The most common organism in men >35 years old is E. coli. In men <35, other causes must be considered including sexually transmitted infection where Chlamydia is one of the commonest infectious agents. Many patients have co-existing orchitis, but in this case the testis is not tender or swollen. Varicoceles (if thrombosed) can present with acute pain however on examination the engorged venous plexus will be visible/palpated which was not present in this patient. Testicular torsion presents over hours with acute onset severe pain, abnormal lying testes and absent of cremaster reflex. Torsion of hydatid of Morgagni presents similar but usually over a longer time frame and can have a classical ‘blue dot sign’. The timeline and physical exam however, do not support these two diagnoses.

95
Q

Treatment of amoebic abscess vs hydatid cyst

A

Amoebic abscess- protazoa -Entomoeba hystolytica * Rx with Metronidazole (antiprotazoal) Hydatid cyst - Echinoccocus (tape worm, parasite) - * Rx with Albendazole (anti Helminthics)

96
Q

Types of benign and malignant parotoid tumours.

A

Salivary glands- benign * pleomorphic adenoma* warthins tumour malignant * adenocystic carcinoma * mucoeitheliod carcinoma * note proportion of malignant tumours increases as the gland gets smaller.

97
Q

Virchow’s node

A

An enlarged and painless supraclavicular lymph node (Virchow) can be a worrying sign, particularly in an older patient. This finding is often the first sign of metastatic spread from an intra-abdominal or intra-thoracic neoplasm Supraclavicular lymph nodesSupraclavicular fossa, closer to the sternal end of the clavicleRight supraclavicular nodes drain into the right lymphatic duct and collect* Neck* Right thorax* Right upper extremityRight mediastinumLungsEsophagusLeft supraclavicular nodes (Virchow nodes) drain into the thoracic duct* Left thoraxLungs* Left upper extremity* AbdomenPelvisLung carcinomaSarcoidosisLeft supraclavicular lymphadenopathy (Troisier sign) is usually indicative of a possible abdominopelvic malignancy (especially metastatic gastric or pancreatic carcinoma).An enlarged left supraclavicular lymph node. Classically associated with metastatic gastric cancer but can develop in other abdominopelvic malignancies (e.g., gallbladder, pancreas, kidneys, testicles, prostate, ovaries).

98
Q

VTE prophylaxis in major orthopaedic surgery.

A

Low molecular weight heparin (LMWH), such as enoxaparin is the preferred option for venous thromboembolism prophylaxis. 40mg is the standard dose given no renal impairment. All adult orthopaedic patients undergoing total hip arthroplasty and knee arthroplasty require thromboprophylaxis. Total hip arthroplasty, requires 28-30 days of LMWH; whereas total knee arthroplasty requires 14 days of LMWH.Unfractionated heparin is an option for patients who have renal impairment and those at an increased of bleeding. Other options, particularly for total hip or knee arthroplasty are direct oral anticoagulants (DOAC), specifically rivaroxaban.Learning PointsMajor surgical procedures will usually require a period of anticoagulation for at least 14 days after discharge from hospital.

99
Q

what are the stages of PAD

A

Up to 20–50% of patients with PAD are asymptomatic.Intermittent claudication* Seen in approx. 10–35% of patientsPain, cramps, or paresthesia distal to arterial occlusionFemoropopliteal disease (most common): typically causes calf claudicationAortoiliac disease (Leriche syndrome)Level of the aortic bifurcation or bilateral occlusion of the iliac arteriesTriad of bilateral buttock, hip, or thigh claudication, erectile dysfunction, and absent/diminished femoral pulsesTibiofibular disease: typically causes foot claudication* Worsens upon exertion ,* Completely relieved by rest or lowering affected limbs* Reproducible on asking the patient to walk the same distance at which symptoms typically occurRest painRest pain occurs as disease progresses and indicates severe ischemia.* Typically occurs first in the toes and forefoot* Worsens with reclining (e.g., while sleeping)* Improves on hanging feet off the bed or on standingCritical limb ischemia (CLI) [3]* Indicative of limb-threatening arterial occlusionCharacterized by the presence of any one of the following:Rest pain lasting ≥ 2 weeksNonhealing ulcersTissue loss (gangrene)Examination findingsTrophic changes↓ Skin temperature, ↓ perspiration↓ Hair on legs, ↓ nail growth, brittle nailsAtrophied musclesDry atrophic, shiny skin and/or bluish skin discolorationLivedo reticularis (advanced disease)Gangrene, ulcers, necrosis (end-stage disease): See also “Arterial ulcer.”Absent or diminished pulses: examine distal pulses bilaterally, includin

100
Q

what does a tinkling bowel sound indicate

A

When the bowel is obstructed they become high pitched or tinkling as fluid drips from one distended and tympanic loop of bowel into another. In the presence of peritonitis, classically in association with a perforation, the abdomen becomes silent reflecting an absence of peristaltic activity, so called paralytic ileus.

101
Q

what is the presentation of acute sinusitis

A

presentation of acute sinusitis can be divided into major and minor symptoms. Major symptoms include purulent nasal discharge, nasal congestion or obstruction, facial congestion, fullness, or pressure, or fever. Minor symptoms include headache, halitosis, and ear pain.

102
Q

what is the strongest indicator for surgery in hyper parathyroidism (ie a parathyroid adenoma)

A

highly resourced health care settings, in which patients are routinely screened, patients with primary hyperparathyroidism typically present with moderate hypercalcaemia and non-suppressed to high PTH levels. Up to 85% of cases are a result of a single parathyroid adenoma. What are the Complications of the disease? * renal calculi, * peptic ulceration,* acute pancreatitis and * fractures.Indications for surgery remain an area of controversy, however patients with complications, or at high risk of developing complications of primary hyperparathyroidism are routinely accepted for surgical intervention. Generally accepted indications for surgery include age < 50 years, * osteoporosis, ie T score less than 2.5 or fragility # (compressive vertebral or pathalogical #) presentcreatinine clearance < 70% of age matched individuals,* hypercalciuria > 10mmol/day, * hypercalcaemia > 2.85 mmol/L and * patients for whom medical surveillance is not possible, as these patients are at high risk of developing complications.Learning PointsOsteoporosis is an important indicator for surgery in cases of primary hyperparathyroidism. What is defined as Osteoporosis T-score is defined as the difference in standard deviations between the patient’s BMD and the BMD of a young adult female reference mean.- Osteoporosis: T-score ≤ -2.5 SD- Osteopenia: T-score of -1 to -2.5 SD Osteoporosis is diagnosed if T-score ≤ -2.5 SD and/or a fragility fracture is present. Fragility fractures: pathological fractures that are caused by everyday-activities (e.g., bending over, sneezing) or minor trauma (e.g. falling from standing height) - Common locations: vertebral (most common) > femoral neck > distal radius (Colles fracture) > other long bones (e.g., humerus)- Vertebral compression fractures- Commonly asymptomatic but may cause acute back pain and possible point tendernes

103
Q

what is the most common organism associated with biliary sepsishow does billary sepsis present

A

Charcot’s triad present of right upper quadrant pain, jaundice, and fever. There is a risk of this progressing to Reynolds’ pentad with additional features of shock and altered mental status. Biliary obstruction is most commonly due to choledocholithiasis, wThe most frequently isolated pathogens are gram-negative enteric organisms: Escherichia coli (25-50%), Klebsiella species (15-20%), and Enterobacter (5-10%); along with gram-positive Enterococcus (10-20%). Anaerobic bacteria such as Bacteroides fragilis and Clostridium perfringens are identified in approximately 15% of cases, and particularly if there has been previous biliary surgery or in the elderly population. This highlights the importance of empiric antibiotic therapy covering for gram-positive, gram-negative and anaerobic organisms.Learning PointsEscherichia coli is the most common organism associated with biliary sepsis

104
Q

when is treatment for decreased bone density indicated

A

Medical therapy [17]IndicationsHistory of fragility fracturesT-scores ≤ -2.5Drug of choiceBisphosphonates: e.g., alendronate, risedronateSide effects [18]HypocalcemiaEsophagitis, esophageal cancerOsteonecrosis of the jawAlternative drugs [9]Teriparatide: parathyroid hormone analogRaloxifene: (selective estrogen receptor modulator, SERM) for patients with contraindications to bisphosphonates or those who also require breast cancer prophylaxis (but increases the risk of thromboembolismDenosumab (monoclonal antibody against RANKL)* Indicated in patients with impaired renal functionHormonal therapy

105
Q

When should X-Ray be taken with a suspected knee joint #

A

A useful tool to utilise in presentations of acute knee injuries is the Ottawa Knee Rules- highly sensitive for detection of fractures. Under these guidelines, a knee x-ray is indicated if: (a patient has any of the following-patient age > 55 yearshead of the fibula tendernessisolated patella tendernessinability to flex knee to 90 degreesinability to bear weight (ie. inability to take four steps) immediately after event and at time of presentation * 55, 4 steps (WB), 90 degs, patella and fibula head Based on the patient’s age, an x-ray of the knee is indicated. Further, considering this patient’s age and the intermittent leg stiffness described, it is quite possible that this patient may have osteoarthritis. Both overuse and osteoarthritis can precipitate joint effusions and seem likely in this presentation.

106
Q

when to refer for aortic aneurysm

A

An abdominal aortic aneurysm (AAA) is the abnormal focal dilatation of the abdominal aorta. Common risk factors for AAA are- smoking (most common),- advanced age (ie, >60 years), - atherosclerosis, male gender and connective tissue diseases. Most cases of AAA are asymptomatic. Patients with symptomatic AAA present with abdominal or back pain, or pain due to distal embolisation. Although many studies have revealed a beneficial effect of screening for AAA, no screening protocol has been established in Australia despite being common in practice in other countries such as the UK. The management of AAA depends on the size, progression, and symptoms of the aneurysm. Patients with an asymptomatic AAA with size <5.5 cm are followed up and advised against smoking. The indications for surgical management of AAA are as follows:* Size >5.5 cm* Growth in size of >1 cm/yearSymptomatic AAA This patient fulfils two of the three indications for surgical repair (ie, size >5.5 cm and symptomatic AAA). The sudden onset of loin pain in the presence of a large AAA suggests that the aneurysm may have leaked or is likely to rupture, so an urgent surgical referral and repair is advised. Learning PointsLarge aneurysms (>5.0 cm) need to be referred for a vascular opinion.

107
Q

with a three month history of cramp-like pain in his right calf. walks more than 300 metres and goes away when he rests. He has a 30 pack-year smoking history and has hypertension and type II diabetes. left leg normal.The right femoral and popliteal pulses also feel normal, but the right dorsals pedis is barely palpable. Power and sensation on both legs appear to be normal, apart from reduced sensation in both feet and weak ankle jerk reflexes. The ankle-brachial index is 1.0 on the left and 0.95 on the right.Which one of the following is the most appropriate next step?

A

Measurement of exercise-induced ankle-brachial index (ABI) is the most appropriate next investigation. The history is very suggestive of intermittent claudication, but the examination findings do not bear this out. Current evidence suggests that, in patients with normal or borderline resting ABIs, the next most appropriate investigation is an exercise-induced ABI. This study is performed immediately following a patient walking on a treadmill at a reasonable pace and gradient (or, if unavailable, walking up and down stairs/a hallway) and assessing their ABI.Duplex ultrasound provides a sensitive and specific (90 and 95%, respectively) means of assessing the location and degree of stenosis. However, as this patient has demonstrated normal ABIs, the next most appropriate investigation is to repeat them following exercise.

108
Q

30-year-old man presents , 12 hour history of gradual onset left-sided testicular pain. There is no history of trauma and the patient is otherwise well, with no fever, malaise or urethral discharge.The left scrotum is swollen, erythematous, and tender on palpation. A cremasteric reflex is present bilaterally and elevating the scrotum relieves the pain initially.most appropriate investigation

A

This patient is most likely presenting with epididymitis with a key differential of testicular torsion.Whilst surgical exploration is of high priority in cases of suspected torsion, if arranged quickly, scrotal Doppler ultrasound can differentiate these two key diagnoses through assessing blood flow to the scrotum and avoid further unnecessary intervention. Epididymitis will typically show increased blood flow relating to acute inflammation, whereas torsion will demonstrate decreased blood flow.If this patient was younger or scrotal ultrasound was not readily available, surgical exploration would be required to exclude testicular torsion given the high risks associated with missed diagnosis.Urine culture and urine NAAT will be helpful in establishing a potential cause for this patient’s epididymo-orchitis, with a sexually transmitted infection such as Chlamydia trachomatis or Neisseria gonorrhoea more likely in a patient of this age group compared to older men where bacterial causes secondary to urinary stasis may be more likely.C-reactive protein may be elevated in many situations including both infection and torsion, and hence will not be helpful in reaching a diagnosis.Learning PointsEpididymitis is a common cause of acute scrotal pain and erythemaDoppler ultrasound is useful in differentiating key causes of scrotal painSurgical exploration should not be delayed in cases of scrotal pain with unclear diagnoses in young patients

109
Q

unable to move the wrist or fingers from the position seen in the image. The GP additionally notes marked atrophy of the forearm on the affected side.He is otherwise well with no significant medical history, and the GP finds no other deformities on examination.

A

Key Clues Wrist- unable to move –forearm musclesFingers- unable to extend (1st 3 dugits)Forearm atrophyInjuries to the median nerve and brachial artery, which both cross the elbow, are common complications of supracondylar fractures.The Median nn and brachial aa travel together over the medial lateral distal shaft over the lateral chondyl of the humerus. Supracondular fractures are the most common # in childhood. Complications of SC#:Volkmann’s ischaemic contracture. This is the result of an acute ischaemic insult to the flexor muscles of the forearm causing permanent fibrosis and shortening. The most common cause is a fracture of the distal humerus - specifically a displaced supracondylar fracture causing either disruption of the brachial artery, or compartment syndrome of the forearm. This is a rare condition which can be treated with physical therapy and surgery. Radial nn more likely to be injured in humeral shaft #.Radial nn wraps around lateral epichondyl then moves posteriorly.Ulna nn- posterior aspect medial epichondyl.

110
Q

VTE risk due to the risk factors related to their reason for hospitalisation ie procedural risk vs risk of bleeding weighed up. Mechanical vsChemmical

A

The following specific patient groups are recognised to be at increased VTE risk due to the risk factors related to their reason for hospitalisation and will require VTE prophylaxis. All other patients that are not covered in the list below will require an individualised VTE risk assessment to determine their VTE risk and need for VTE prophylaxis.Medical patients• Admitted following acute stroke with significant reduction in mobility• Critically ill patients• Hospitalised for decompensated heart failure• Active inflammatory bowel diseaseSurgical patients• Major abdominal-pelvic surgery for cancer• Total hip arthroplasty or total knee arthroplasty• Fragility fractures of the pelvis, hip and proximal femur• Major trauma surgery (including traumatic brain injury, acute spinal cord injury, traumatic spinal injury and complex traumatic pelvic / lower extremity injury)• Craniotomy• Cardiac surgery• Abdominal aortic aneurysm repair surgery• Thoracic surgery patients with primary or metastatic cancer• Elective spinal surgery (with hospital admission) resulting in reduced mobility• Bariatric surgery Ambulatory patients with isolated lower limb immobilisation• Ambulatory patients temporarily immobilised with above or below knee cast, or backslab with additional risk factors

111
Q

Exercise treadmill ABI

A

Exercise treadmill ABI is performed for patients with intermittent claudication and a borderline ABI of 0.9. A post-exercise ankle SBP decrease >30 mmHg or a post-exercise ABI decrease >20% are diagnostic for PAD.

112
Q

Patients with a ruptured AAA

A

Patients with a ruptured AAA may initially present with no symptoms, but most (75%) will present with pain, which is typically located in the abdomen and radiating to the groin but may also be in the back, flank or pelvis. Signs of shock may also be present with the classic triad of pain, pulsatile abdominal mass and hypotension present in 50% of patients with ruptured AAA.

Diagnosis of AAA depends on haemodynamic status. In unstable patients with a suspected ruptured AAA imaging is desirable but not essential prior to management. Bedside ultrasound may be used in this scenario to identify the AAA. In stable patients, urgent abdominal CT with contrast is preferable for diagnosis.

Management of ruptured AAA involves emergency and immediate control of haemorrhage in theatre with laparotomy followed by repair of the aneurysm.

Learning Points
Leaking aneurysms require rapid resuscitation and intervention.

113
Q

VTE for Total hip arthroplasty, and total knee arthroplasty

A

hip hemiarthroplasty requires 28-30 days of LMWH; whereas total knee arthroplasty requires 14 days of LMWH.

Unfractionated heparin is an option for patients who have renal impairment and those at an increased on bleeding. Other options, particularly for total hip or knee arthroplasty are direct oral anticoagulants (DOAC), specifically rivaroxaban.

Learning Points
Major surgical procedures will usually require a period of anticoagulation for at least 14 days after discharge from hospital.

114
Q

signs of intestinal obstruction, with pnuemobillia.

possible cuases

cholangitis (no fever,RUQ pain, jaundice, chollithiasis, cholangiocarcinoma ( no LOW, not jaundiced), liver abcssess, incisinal hernia ( never had surgery)

A

clinical and radiological features consistent with a small bowel obstruction, including central colicky pain, early bilious vomiting, obstipation, abdominal distension and tinkling bowel sounds, with dilated loops of small bowel on the X-ray.

gas in the biliary tree (pneumobilia) suggests either infection by gas-forming organisms in the portal venous system and/or free communication of the biliary tree with the gut. This is a picture seen with cholangitis; liver abscess with communication to the biliary tree; spontaneous biliary-enteric fistula from gallstone ileus or cholangiocarcinoma; and, most commonly, as a result of recent biliary instrumentation with endoscopic retrograde cholangiopancreatography (ERCP) or sphincterotomy.

if liver abscess were the culprit, the patient would be expected to have right upper quadrant pain, jaundice, unintentional weight loss and more prolonged nausea and anorexia.

cholangitis, which is characterised by the present of Charcot’s triad: fever, jaundice and right upper quadrant pain,

With cholangiocarcinoma, the clinical picture would include jaundice, weight loss and anorexia

small bowel obstruction with a communication to the bowel. The most likely cause is therefore gallstone ileus. ie the underlying problem has been cholelithiasis. There has been a fistulous communication between the gallbladder and the gut, with an eroding gallstone passing into the small bowel, where it has become obstructed. Air from the gut can enter the biliary tree through the cholecyst-duodenal fistula. A CT scan would help distinguish portal venous gas from pneumobilia.

Learning Points
Gallstone ileus is an uncommon, but well recognised cause of small bowel obstruction.

115
Q

A 76-year-old man undergoes a laparoscopic repair of an incarcerated para-oesophageal hernia. The procedure is uncomplicated. Twelve hours later he feels well and is sitting up in bed. His blood pressure is 100/70 mmHg and his urine output since operation has been 100 ml. He is afebrile. He has intravenous isotonic saline running at 80 ml/hr. His sodium is 140mEq/L, and his potassium is 4.3mEq/L.

Which one of the following is the most appropriate next step in management?
Choose the single best answer.

Continue current management
Intravenous furosemide
One litre Hartmann’s solution bolus infusion
Intravenous ephedrine
One litre isotonic saline over four hours

A

The correct answer is one litre isotonic saline over four hours. This patient is hypotensive and likely fluid-deplete. Whilst some of his hypotension could be due to vasodilatation associated with his recent anaesthetic, 100mL over 12 hours is relatively poor urine output, and this must be corrected. As his potassium is close to the upper reference range (of 4.5mEq/L), Hartmann’s solution should be avoided, as it contains potassium and could precipitate hyperkalaemia.

Ephedrine is the first-line treatment of intraoperative hypotension during general anesthesia. This is unlikely to be a major contributor to his hypotension at this stage post-operatively, and therefore this is inappropriate.

As he is hypovolaemic, a diuretic, or continuing current management would be inappropriate.

Learning Points
Post-operative hypotension is often due to inadequate fluid replacement.

116
Q

post operative hyponatreamia. How to investigate?

morning cortisol (prim hypercort)

urine osalalty vs urine sodium

serum osmolality

A

Postoperative hyponatraemia is a relatively common finding in the post-operative patient and can be due to a combination of fluid retention (secondary to non-osmotic ADH release) and infusion of hypotonic intravenous fluids. Another important cause of hyponatraemia is increased hypertonicity, with a dilutional effect brought about by a shift of water from the intracellular to the extracellular compartment. An elevated blood glucose concentration can do this. While all of the investigations are appropriate in the work-up of hyponatraemia, it is first important to check for hyperglycaemia and calculate the corrected sodium. As the patient is diabetic and has recently had surgery, it would not be unusual for his serum glucose to be high, causing an osmotic water shift from the intracellular to extracellular space and a resultant dilutional hyponatraemia.

Learning Points
Regular blood sugar monitoring is essential in patients with diabetes who are acutely ill.

117
Q

A 60-year-old man presents with a two month history of a swelling in the left side of his scrotum. It is only visible when he stands. The swelling is soft, non-tender, and is above the left testis. It can be felt distinct from the left testis. There is no cough impulse and the swelling appears to extend up into the inguinal canal. It cannot be reduced into the abdominal cavity.

Which one of the following is the most likely diagnosis?

A

This is a textbook description of a varicocele - ie a dilatation of the veins of the pampiniform plexus. In other words, these varicosities can extend right up into the inguinal canal.

An inguinal hernia would have a cough impulse and unless very large in size, can usually be reduced to within the abdominal cavity.

A hydrocele envelopes the testis - and if large and tense, the testis cannot be felt.

An epididymal cyst is attached to the testis and when present, can be palpated posteriorly or at the upper pole of the testis.

An hydatid of Morgagni is a common vestigial remnant and is attached to the lower pole of the testis.

Learning Points
Varicoceles are very common and usually of no clinical significance.

118
Q

Asymptomatic carotid artery stenosis of <50% stenosis can be treated with:

  • anti-platelets and risk factor modification alone.
  • revascuralisation with endaretectomy ?
A

Perform carotid artery imaging in all patients with symptomatic carotid stenosis.

  • Clinically significant carotid stenosis: narrowing of the carotid artery ≥ 50%
  • Moderate carotid stenosis: narrowing of the carotid artery by 50%–69%
  • Severe carotid stenosis: narrowing of the carotid artery 70%–99%

For severe symptomatic carotid stenosis.

Balloon angioplasty, carotid stent and carotid endarterectomy are all surgical interventions to manage carotid stenosis.

When indicated, carotid endarterectomy is the preferred of the three surgical procedures. It is indicated in symptomatic patients (TIA or stroke) with stenosis of 70-99% and those with symptomatic stenosis of 50-69% who are low surgical risks.

Anti-platelets and risk factor modification are recommended in symptomatic patients with <50% stenosis. Patients with asymptomatic carotid artery stenosis should be treated with aspirin, and if they have an aspirin allergy, they should be treated with clopidogrel. In addition, risk factor modification plays an important role. The management of hypertension, dyslipidaemia, diabetes, and smoking is also recommended.

For endarectemy

  • Periprocedural risk and patient life-expectancy, comorbidities, and preferences must also be considered.
  • Contraindications [3][18]
    • Carotid stenosis < 50%
    • Chronic complete carotid occlusion
    • Severely disabling stroke
119
Q

Primary small bowel tumours are an uncommon, but important cause of small bowel obstruction.

A 62-year-old man presents with a three week history of nausea, vomiting and post-prandial abdominal distension. He has not had any other gastrointestinal symptoms. The physical examination is unremarkable. On investigation he is found to have a tumour obstructing his proximal jejunum. This is resected and found to be a primary adenocarcinoma.

Which one of the following risk factors is most likely to be present?
Choose the single best answer.

Familial adenomatous polyposis
Peutz-Jegher syndrome
Coeliac disease
Ulcerative colitis
Crohn disease

A

Familial adenomatous polyposis (FAP) is the most likely risk factor present in this patient to cause him to have an adenocarcinoma in his bowel.

FAP is caused by an autosomal dominant mutation in the APC gene, which is a tumour suppressor gene located on chromosome 5q22.

Patients with FAP typically develop polyposis (widespread bowel polyps) in their second or third decade of life, with them most likely remaining asymptomatic until progressing to colorectal cancer. This is consistent with the presentation of this patient. These patients can have up to thousands of colorectal adenomas and a 100% risk of colorectal cancer in their lives.

Patients with FAP and patients with the Lynch syndrome have an increased risk of developing small bowel cancer, with the lifetime risk being about 5%.

Peutz-Jeghers syndrome is another risk factor developing colorectal cancer, due to an autosomal dominant mutation in the STK11 gene. These patients typically develop hyperpigmented macules around their lips, genitals, palms, soles, and buccal mucosa – making this diagnosis less likely than FAP, as the patient does not have any of these overt signs.

Coeliac disease is an autoimmune disorder characterised by an intestinal hypersensitivity to gluten. A complication of this is a moderately increased risk of malignancies, such as enteropathy-associated T-cell lymphoma (EATL), often located in the proximal jejunum. This is not consistent with the resection being a primary adenocarcinoma in this patient.

Crohn’s disease is associated with increased risk of colorectal cancer. These patients are also at risk of developing small bowel tumours, but the tumours tend to be in the distal small bowel. In this case, the clinical presentation is not consistent with Crohn’s disease.

Ulcerative colitis causes an increased risk of colorectal cancer. However, it does not cause an increased risk of upper gastrointestinal tumours, as ulcerative colitis impacts the rectum and can extend in a proximal and continuous fashion to involve other parts of the colon.

120
Q

A 70-year-old woman presents with upper abdominal and left shoulder tip pain one day after a colonoscopy. At the procedure a carcinoma of the caecum was identified and biopsied. On examination her blood pressure is 110/70 mmHg, pulse 105/min and temperature 37.2C. She is tender in the epigastrium. The only abnormality on her laboratory investigations is a haemoglobin of 93gm/L (115-155). A CT scan is performed.

Which one of the following is the most likely diagnosis?
Choose the single best answer.

Acute pancreatitis

Splenic laceration
Postpolypectomy syndrome
Colonic haemorrhage
Perforation of the colon

A

The symptoms and physical signs add up to peritoneal and diaphragmatic irritation - and the CT shows a rupture of the spleen.

This is a rare, but well-recognised complication of colonoscopy. More frequently encountered complications include bleeding and perforation. Neither of these complications are common.

Bleeding may occur in up to 6% of cases after polypectomy, but less than 1% of cases will require any form of intervention.

Perforation can be more serious, but again, it is an uncommon occurrence, being seen in less than 1% of diagnostic investigations and 5% of therapeutic procedures.

Postpolypectomy syndrome occurs when there is an area of localised perforation following a polypectomy, where the damage has been brought secondary to diathermy-induced tissue necrosis. The perforation may occur immediately at the time of the procedure and later, when the necrotic tissue sloughs.

In this case the CT imaging clearly demonstrates a splenic injury and the investigations and physical examination findings are consistent with acute haemorrhage.

Learning Points
Abdominal pain after a colonoscopy must always raise the possibility of a complication of the procedure.

121
Q

Most foreign bodies in adults pass without any need for intervention.
High-risk foreign bodies may require endoscopy.

What is high risk

A

Endoscopy is required in high-risk presentations of foreign body ingestion. This includes patients with evidence of complete oesophageal obstruction, objects larger than 2.5cm and still in the stomach, and objects larger than 6cm in the proximal duodenum or above.

Button batteries pose a risk of erosion and perforation. If swallowed and recognised, they should be retrieved wherever possible. Sharp objects only require endoscopy if they are in the proximal duodenum or above; as this is when the greatest risk for perforation is, particularly as the object passes through the pylorus.

Although colonoscopy can be used for extremely high risk, large, foreign bodies that are in the colon, this is not the case in this patient as this foreign body will almost certainly pass without the need for intervention.

In this patient’s X-ray, there is a normal gas pattern in the colon and the foreign body can be seen immediately to the left of the second lumbar vertebra and is almost certainly in the transverse colon (Image). Given his lack of symptoms and signs of perforation, the location of the foreign body, and the time since ingestion, there is a very high chance it will pass spontaneously and no intervention will be required, but observation and review are necessary; particularly a psychiatric screen for self-harm and suicidality.

CT abdomen is not required as the location of the foreign body is clear in the x-ray and clinical presentation. If the object had multiple parts, was difficult to see on x-ray, or there was any doubt about its location, a CT scan may be justified.

Laparoscopy is reserved for patients who have signs of perforation in foreign bodies.

122
Q

On examination she looks ill, with a blood pressure of 110/70 mmHg, pulse 120/min and temperature 37.4 C. Her abdomen is diffusely tender. A CT of the abdomen is performed (Images 1, 2).

Which one of the following is the most likely diagnosis?

A

The scan shows gas in the portal vein and areas of pneumatosis intestinalis in the mid small bowel. Taken together with the clinical picture, these radiological findings would fit for a diagnosis of mesenteric ischaemia with probable bowel infarction. Hepatic portal venous gas can have several different origins, including diverticulitis, gastroparesis, appendicitis and acute pancreatitis. However, these causes are uncommon and the most usual underlying problem is bowel ischaemia and/or infarction. This radiological finding is associated with a high mortality rate.

Learning Points
Gas in the portal vein must always raise the possible diagnosis of mesenteric ischaemia.

123
Q

Ischeamic collitis vs mesenteric ischeamia

A

The most likely diagnosis is ischaemic colitis, making a contrast-enhanced CT scan the investigation of choice. Evidence of colonic wall thickening, pneumatosis and mesenteric stranding should be sought. CT has the added advantage of being able to identify other possible causes of the current problem, should the diagnosis not be ischaemic colitis.

Mesenteric angiography would be most appropriate with suspected mesenteric ischaemia. Angiography allows direct visualisation of mesenteric vascular occlusion, the extent of the lesion which may be causing it and any possible therapeutic interventions.

124
Q

A 72-year-old woman presents with a 6 hour history of rapid-onset, colicky abdominal pain associated with the passage of loose, bloody stool. She has a history of hypertension and hypercholesterolaemia. Her regular medications include perindopril, simvastatin, and aspirin. On examination, she is in severe pain with a blood pressure of 140/90mmHg, an irregular pulse rate of 100/min and temperature of 37.4. There is some left sided abdominal tenderness and the rest of the abdomen is soft.

Which one of the following investigations is the most appropriate next step?

Plain abdominal X-ray
Contrast-enhanced CT abdomen
Mesenteric angiography
Flexible sigmoidoscopy
Colonoscopy

A

Learning Points
Contrast-enhanced CT scans are the investigation of choice in ischaemic colitis

The most likely diagnosis is ischaemic colitis, making a contrast-enhanced CT scan the investigation of choice. Evidence of colonic wall thickening, pneumatosis and mesenteric stranding should be sought. CT has the added advantage of being able to identify other possible causes of the current problem, should the diagnosis not be ischaemic colitis.

A plain abdominal X-ray would be appropriate in a suspected bowel obstruction, wherein it would demonstrate dilatation of bowel loops and air-fluid levels.

Mesenteric angiography would be most appropriate with suspected mesenteric ischaemia. Angiography allows direct visualisation of mesenteric vascular occlusion, the extent of the lesion which may be causing it and any possible therapeutic interventions.

Flexible sigmoidoscopy is useful for the diagnosis of lower gastrointestinal conditions, such as diverticulosis or malignancies, which will demonstrate the macroscopic apperance of the colonic mucosa.

Colonoscopy are useful for similar reasons to flexible sigmoidoscopy, although extend further through the colon, and should be performed within 48 hours to exclude other types of colitis (tissue biopsies may be required). However, contrast-enhanced CT is a more appropriate in the setting of suspected ischaemic colitis.

125
Q

How to decide - bcc , hydatid cyst , liver abscess

A

Hydatid disease used to be common disease in sheep-rearing areas in Australia, where man became an inadvertent secondary host, after becoming infected from the primary hosts - dogs. Deworming of pets and working dogs has lowered the incidence of the disease in Australia, although hydatid remains relatively common in some parts of the world, such as Turkey. In endemic regions the prevalence of hydatid varies between 1% - 10% of the population. Most cases of hydatid cyst are incidental findings and have a characteristic calcified rim. In addition, the contents of a hydatid cyst tend to be homogenous - apart from the daughter cysts and ‘hydatid sand.’ This patient has a typical hydatid cyst with visible daughter cysts. Patients with liver abscess commonly present with infective symptoms including fever and malaise. On imaging, liver abscess present as a solid, single or multilocular cavity with possible gas collection. Acute cholecystitis is a less likely diagnosis due to the duration of the symptoms. Patients with acute cholecystitis usually present with acute severe right upper quadrant abdominal pain. It is also unlikely to diagnose acute cholecystitis with this film as the slice does not feature the gallbladder/gallbladder fossa. Liver lesions due to metastatic liver disease present as multiple lobulated solid lesions with possible central necrosis. Hepatocellular carcinoma clinically presents with jaundice, ascites, and constitutional symptoms of anorexia and weight loss. On CT imaging, hepatocellular carcinoma lesions is characteristically more hypervascular and solid than cystic. Learning Points Contents of a hydatid cyst tend to be homogenous - apart from the daughter cysts and ‘hydatid sand

126
Q

Patient with recent Crohns diagnosis.; A colonoscopy performed two months ago found extensive shallow ulceration in the colon and distal ileum. Magnetic resonance imaging showed one additional segment of the ileum affected and no evidence of perianal fistula. Treatment was started with prednisone and this provided good symptomatic relief over the course of three months. Which one of the following initial treatment options would be most appropriate? Choose the single best answer. Azathiaprine Mesalamine Continue corticosteroid Infliximab Ustekinumab

A

This patient has been treated with corticosteroids for an acute flare-up of Crohn’s disease. Thus, they should be initiated on maintenance therapy to prevent further relapses of Crohn’s disease. This is done first line with azathioprine. In practice, a patient like the one described in this scenario, will get infliximab fairly rapidly after starting azathioprine. Infliximab is considered second-line therapy, and is added in addition to azathioprine if monotherapy alone fails. Data is appearing in the literature suggesting that patients who are started early on infliximab (<2 years after diagnosis) have better endoscopic remission rates and less stricture formation compared with those are started later (>2 years after diagnosis). Mesalamine is not recommended in the maintenance/prevention of relapse in Crohn’s. Although corticosteroids are useful in acute flare-ups, they are not recommended in long-term prevention of relapse in Crohn’s disease. Ustekinumab is considered third-line therapy, and is added if the addition of an anti-TNF agent such as infliximab fails to prevent relapse. Learning Points Azathioprine therapy is considered first-line in Crohn’s disease maintenance of remission, to prevent further relapses.

127
Q

Prostate Ca Mx

63-year-old man is referred to the urology outpatient clinic with a two month history of increasing back pain and a PSA of 37.1 ng/mL. His medical history includes asthma and psoriasis. On examination he is tender over his lumbar vertebrae and on digital rectal examination, his prostate feels nodular and firm. Anal tone is preserved and there is no perianal numbness. His lower limb neurological examination is normal. A 99m-technetium whole body bone scan shows increased uptake in the lumbar and lower thoracic vertebrae.

Aside from pain relief, which one of the following would be the most appropriate treatment?

Choose the single best answer.

  • LHRH antagonist
  • Non-steroidal anti-androgen
  • LHRH agonist
  • 5-alpha reductase inhibitor
  • Corticosteroid
A
  • LHRH antagonist- drug of choice for immediate castration (2-3 days), reduced risk of a tumor flare - especially important in patients with spinal metastasis.
  • LHRH agonist- initial spike of testosterone can drive tumour flare
  • Non-steroidal anti-androgen- not monotherapy used with LHRH agonist to reduce tumour burden
  • Corticosteroid- adjunct after chemotherapy
  • 5-alpha reductase inhibitor- used in BPH treatment!!!

This patient presents with metastatic prostate cancer. This disease is incurable and treatment must be focussed on symptomatic control. Some form of androgen deprivation therapy would be appropriate. The choice of androgen deprivation in this case is based on the mechanism of action and the subsequent risk of causing a tumour flare.

The most a**ppropriate choice of treatment in this scenario is an LHRH antagonist**. Unlike LHRH agonists, **this particular drug class is not associated with a tumour flare**. Castrate levels of **serum testosterone will be reached within 2-3 days**. 
**Degarelix is an example of an LHRH antagonist.**

LHRH agonists acts on the pituitary gland, initially increasing the serum testosterone transiently before causing it to fall to castrate levels. This starts 2-3 days after initial administration and lasts for approximately one week. In patients with symptomatic metastatic disease there is a risk of causing worsening local symptoms. With spinal metastases this means a heightened risk of pathological fractures or malignant spinal cord compression. Given that the bone scan demonstrates malignant spread to the vertebral column, LHRH agonist is not the best answer.

Agents of the non-steroidal anti-androgen class are sometimes given at the same time as LHRH agonists, in order to prevent the tumour flare. As such, the role of this drug class is synergistic-these medications should not be given as monotherapy as they have reduced overall survival when compared with LHRH agonist/antagonists. The most appropriate choice of treatment in this scenario would be degarelix, a LHRH antagonist that is not associated with a tumour flare and leads to castrate levels of serum testosterone within 2-3 days.

The 5-alpha reductase inhibitor drug class is not the best answer option. Whilst these medications have a proven and valuable role in the treatment of benign prostatic enlargement and lessening the troubling symptoms of this condition, their value in the management of prostate cancer is far more uncertain. For metastatic disease, they are likely to be of little benefit.

Corticosteroids are widely used in the setting of metastatic castrate-resistant prostate cancer, often in combination with chemotherapeutic agents. Their anti-inflammatory properties also makes them useful in the palliative care setting. Whilst this patient does indeed have metastatic disease, it is more appropriate to attempt chemical castration to lessen progression of disease before moving onto alternative therapies such as corticosteroids. As such, corticosteroids is not the correct answer.

Learning Points
Choice of androgen deprivation is based on the mechanism of action and risk of tumour flare.

128
Q

Prostate Ca Mx

  1. What are common drug names
    1. LHRH antagonist
    2. LHRH agonist
    3. Non-steroidal anti-androgen
A
  1. Androgen deprivation therapy
    1. LHRH antagonist (GNRH atagonists) - no tumour flair: metatstatic Prostate Ca, e.g., degarelix
    2. LHRH agonist (GNRH agonists) e.g., leuprolide, goserelin,
    3. GnRH receptor antagonist (e.g., relugolix)
  2. Androgen synthesis inhibitors / Andrgen Receptor Antagonist
    1. Non-steroidal anti-androgen- (Andrgen Receptor Antagonist)
      apalutamide and enzalutamide (second-generation, stronger Receptor affinity)- adjunct to LHRH agonists ( luperilide and goserelin)
    2. ASI- inhibits glucocorticoids, increases adrenal mineralocorticoids- Commonly used agent: abiraterone
129
Q

Imaging and obstruction

78-year-old woman presents

four week history of increasing constipation associated with crampy abdominal pain and distension.

HX appendicectomy and an hysterectomy.

On examination her blood pressure is 134/84 mmHg, pulse 90/min and temperature 37.2C. Her abdomen is uniformly distended and there is mild widespread tenderness. The bowel sounds are increased and high-pitched. Her rectum is empty and nothing abnormal can be felt on digital examination. An abdominal X-ray showing large bowl dilation, with a clear transitional point at the sigmoid colon with no air in the rectum

A

An unexplained change in bowel habit in an older patient should always raise the possibility of malignancy.

Adhesional obstruction and volvulus tend to be acute events, with symptoms usually only appearing a day or two at most before the increase in their severity leads the patient to seek medical help.

Pseudo-obstruction usually has a clearly defined precipitating cause such as recent surgery or retroperitoneal trauma.

This particular patient has a recent change in bowel habit, suggesting something causing progressive luminal occlusion.
In her age group, colonic malignancy must be excluded and is the most likely explanation for her symptoms.
The diagnosis is given away by the radiological pattern of gas distribution - distended colon with a cut-off in the region of the sigmoid colon and no gas in the rectum. Whilst non-malignant sigmoid lesions or strictures such as in severe diverticulitis could present in such a way, this patient most likely has a carcinoma of the sigmoid colon.

130
Q

types of hypersensitivity reactions

  • Antibody-mediated immune reaction
  • Antigen-antibody complex formation
  • Complement-mediated cytotoxicity
  • IgE-mediated mast cell degranulation
  • CD8+ lymphocyte-mediated hypersensitivity
A
  • IgE-mediated mast cell degranulation- T1 Anaphalaxis
  • Antibody-mediated immune reaction- T2 antigen on APC→ antibodies
  • Antigen-antibody complex formation T3 ( SLE/Lupus nephritus and serum sickness)
  • CD8+ T lymphocyte-mediated hypersensitivity- T4 delayed hypersensitivity reaction and does not present as acutely (poison Ivy)
  • Complement-mediated cytotoxicity- IgG and IGM- Complement-mediated cytotoxicity is caused by IgM and IgG antibodies, not IgE antibodies (anaphalaxis)

Anaphylaxis is a type I hypersensitivity reaction characterized by IgE-mediated cell degranulation. On initial exposure to an antigen, the antigen stimulates antibody class switching to IgE antibodies. These IgE antibodies bind to IgE receptors present on the surface of basophils in the blood and mast cells in the tissues. On subsequent exposure, the antigen binds to the IgE antibodies present on the surface of basophils and mast cells and cause aggregation of the receptors by cross-linking of antibodies. This leads to degranulation and release of histamine and other inflammatory cytokines that are implicated in vasodilation, bronchoconstriction, vascular permeability, and other features seen in anaphylaxis.

Antibody-mediated immune reaction is a type II hypersensitivity reaction that is characterised by antibody binding to the antigen being presented by antigen-presenting cells. After binding, these antibodies release several enzymes that lead to cell lysis and death.

Antigen-antibody complex formation is a type III hypersensitivity reaction that is seen in systemic lupus erythematosus or serum sickness. It is not associated with anaphylaxis.

Complement-mediated cytotoxicity is caused by IgM and IgG antibodies, not IgE antibodies. It is not seen in anaphylaxis.

CD8+ T-lymphocyte-mediated cytotoxicity is a type IV delayed hypersensitivity reaction and does not present as acutely as seen in this patient. One common example of type IV hypersensitivity reaction is poison ivy hypersensitivity reaction.

131
Q

appendicitis

five-day history of lower abdominal pain. The pain was initially intermittent and has now become constant. He has no other symptoms apart from some intermittent nausea.

On examination, the patient’s blood pressure is 115/90 mmHg, pulse 100/min and temperature 37.8ºC. Abdominal examination shows tenderness in the right iliac fossa and no other abnormalities.

reviously well 39-year-old man presents to the Emergency Department with a two-day history of colicky abdominal pain, worse on the right side. The pain started intermittently but has become more severe and constant in the last few hours. He also reports a fever and some nausea. The patient has never been hospitalised and has no surgical history.

On examination, his temperature is 38.4ºC, blood pressure 124/90 mmHg and pulse 90/min. His BMI is 35. Abdominal examination shows an area of tenderness in the right lower quadrant.

A
  • Broad-spectrum antibiotics should be first line management for patients presenting with an appendix mass.

first instance he should be treated with antibiotics. The history and examination are also typical of appendiceal pathology. Acute appendicitis typically presents with abdominal pain that increases in severity, frequency and localises to the right lower quadrant over days.

In most instances, the administration of broad-spectrum antibiotics will resolve the situation and an elective appendicectomy would be considered.

Q2

he clinical description fits well for a diagnosis of acute appendicitis and is supported by a lack of previous surgeries indicating no past history of appendicitis with appendicectomy. Increasing severity and localisation to the right lower quadrant is typical of this diagnosis. Furthermore, the patient’s obesity might explain the paucity of signs on physical examination. The abdominal X-ray shows a calcified faecolith in the right lower quadrant which may have caused this episode of appendicitis.

A score ≤ 2 indicates a very low likelihood of acute appendicitis.

A score ≥ 9 indicates a very high likelihood of acute appendicitis.

132
Q

a dull, burning, retrosternal chest pain and regurgitation of food. He describes the pain often being present after eating, and finds it difficult to lie down for a few hours after eating because it makes the pain worse. He has noticed that it is present when he is consuming both solids and liquids. He also describes an 8-year history of dysphagia to both solids and liquids, which more recently has progressed to regurgitation of solid food. This occurs once or twice a month, and has been present for the past 3 years. It has worsened in the past few weeks, prompting him to seek medical advice. He has not noticed any abdominal pain, changes in bowel motions, or weight loss.

He is otherwise well other than having well-controlled high blood pressure, for which he takes ramipril. He does not smoke or drink alcohol.

A barium swallow study is ordered

A

This patient has end-stage achalasia with severe dilatation of his oesophagus on a background of a 4-year history of dysphagia. Current guidelines recommend cardiomyotomy with fundoplication for patients with severe achalasia.

Botulinium toxin injection is first-line treatment for achalasia in patients that are unfit for surgical intervention; however, this patient is otherwise well and surgically fit.

Pneumatic dilatation is an accepted initial treatment for achalasia, but would hardly be acceptable in this case of end-stage disease.

Stents are currently not recommended for the treatment of achalasia.

Oesophagectomy is reserved for patients with end-stage achalasia who fail other interventions such as cardiomyotomy or pneumatic dilatation. It is very likely that at some stage this patient will benefit from an oesophagectomy.

133
Q

Shoulder dislocation

The right shoulder is abducted and externally rotated. His right elbow remains flexed and forearm internally rotated, supported by the other hand. There is no loss of skin integrity on his right shoulder, but the acromion appears prominent as compared to the left shoulder. His vital signs are stable, and he has a normal radial pulse on both arms.

What is the most likely area of sensory loss in his right upper limb as a result of this injury?
Choose the single best answer.

  • Medial aspect of his right upper arm
  • Lateral aspect of his right upper arm
  • Medial aspect of his right forearm
  • Lateral aspect of his right forearm
  • Dorsal aspect of his right hand
A
  • Medial aspect of his right upper arm-
  • Lateral aspect of his right upper arm- axillary nerve
  • Medial aspect of his right forearm- medial cutaneous
  • Lateral aspect of his right forearm-
  • Dorsal aspect of his right hand- radial nerve

This patient presents with shoulder pain and a shoulder that is abducted and externally rotated with a prominent acromion. This is consistent with an anterior dislocation of the shoulder, commonly due to an anteriorly directed force on the arm as the shoulder is abducted and externally rotated.

Damage to the axillary nerve is a complication that may result from a shoulder dislocation as the nerve courses medially to the surgical neck of the humerus. This may affect the sensory innervation of the lateral aspect of the upper arm, as the posterior terminal branch of the axillary nerve innervates the upper lateral cutaneous nerve of the arm.

The sensory innervation of the medial aspect of the upper arm comes from the intercostobrachial and medial cutaneous nerve.

The sensory innervation of the lateral aspect of the forearm comes from the musculocutaneous nerve,

and the medial aspect of the forearm is from the medial antebrachial cutaneous nerve.

The radial nerve predominantly innervates the dorsal aspect of the hand. The ulnar nerve also innervates the dorsal aspect of the little finger and ½ of the ring finger.

134
Q

81-year-old man is referred to a specialist clinic with a two-year history of difficulty in swallowing. His symptoms have become more noticeable over the last six months. He has no trouble starting to swallow his food, but it is now associated with a cough and episodes of choking. He coughs up pieces of undigested food and tablets that he has recently taken. He has lost five kilograms in the last six months.

A

history alone, it is clear that this patient has oesophageal dysphagia. His weight loss should always raise the possibility of malignancy, but a two year history would be unusual. Coughing up particles of food and other small materials such as tablets is very suggestive that the material is getting caught in the upper oesophagus and being inadvertently inhaled. This clinical picture would fit very well for a pharyngeal pouch (Zenker’s diverticulum). Whilst these diverticula can be visualised with endoscopy, a much more accurate assessment is made with a contrast study (barium swallow). If the presumed diagnosis was one of oesophageal malignancy, then an endoscopy would be the most appropriate first-line investigation. Apart from viewing the lumen of the upper digestive tract, any suspicious lesion could be biopsied. However, in this case the length of the history would make malignancy a very unusual diagnosis.

135
Q

PE

35-year-old woman presents to the Emergency Department with a 12-hour history of nonproductive cough and right-sided chest pain. The pain came on gradually and is worse when she coughs. She had a laparoscopic cholecystectomy one week earlier and was back at work three days later. On examination, she looks well, with a blood pressure of 125/80 mmHg, pulse 90/min and temperature 37.6 C. Pulmonary auscultation findings and the rest of the physical examination is unremarkable. Routine bloods are normal. A D-dimer is 0.4 mg/L.

Which one of the following would be the most appropriate plan of management?
Choose the single best answer.

Reassurance

Chest X-ray

V-Q scan

CT pulmonary angiogram

Doppler ultrasonography

A

Positive for D-dimer>500ng/dl or 0/5mg/L PE likely

This patient with nonproductive cough, normal vitals, low-grade fever, and chest pain most likely has an acute chest infection (eg, acute bronchitis). Although chest pain with a recent history of surgery should alert the physician for the possibility of pulmonary embolism, this patient has a low pretest probability of pulmonary embolism according to Well’s score. Patients with a Well’s score of ≤4 have a low probability of pulmonary embolism, whereas the patients with a Well’s core of >4 have a high probability of pulmonary embolism. Patients with a low pretest probability of pulmonary embolism should be followed up with a D-dimer assay. A D-dimer level of ≤0.5 mg/L excludes the diagnosis of pulmonary embolism. This patient’s Well’s score of 1.5 (due to recent history of surgery) and a D-dimer level of 0.4 mg/L excludes pulmonary diagnosis and should be reassured and managed supportively.

Acute bronchitis is primarily a clinical diagnosis. Chest X-ray would be indicated if this patient had features of pneumonia (eg, high-grade fever, crackles, productive cough with yellow sputum).

V-Q scan and CT pulmonary angiogram are indicated in patients with a high pretest probability of pulmonary embolism. CT pulmonary angiogram is preferred but V-Q scan may be used if the patient has contraindications to CT pulmonary angiogram (eg, renal failure) or if CT pulmonary angiogram is unavailable.

Doppler ultrasonography of the affected limb may be performed if the patient has clinical features suggesting deep venous thrombosis (DVT). This patient does not have any feature suggesting a DVT.